SlideShare una empresa de Scribd logo
1 de 152
Fayza Rayes
MBBCh. MSc. MRCGP
Consultant Family Physician
Joint Program of Family & Community Medicine, Jeddah
www.fayzarayes.com
Objectives:
1. Improving your out come from studding
2. Improving your technique in answering MCQs
Introduction:
1. Do you follow any techniques while studying from
MCQ book
2. Do you follow any techniques while answering
MCQs questions
Contents:
 10 tips in studying for MCQs exam
 10 tips in answering MCQs
 Answering 45 MCQs
10 Tips in studying MCQs:
1. Write your studying plan
2. Study in a group
3. Answering one MCQ is an opportunity to review lots of
knowledge . Make sure to get the maximum from every
single MCQ . Read the explanation and review the
related topic
4. Write a short abstract about each important topic
5. Write table to compare between similar diseases
6. Be specific in your information and extra specific
How can you benefit from this information ?
10 Tips in studying MCQs:
7. Concentrate on important topics in family medicine e.g.:
 Children growth and development
 Vaccination
 Presentation of common diseases special patients. e.g.
appendicitis in very young and in very elderly patients
 Prescribing skills of antibiotics and psychotropic
medications
10 Tips in studying MCQs:
8. To improve your mental fitness, periodically take mock
exam, start by 10 questions then increase it gradually till
you take full mock exam (100-150 MCQs in 2-3 hours
continuously)
9. Revise your mistake periodically (some mistakes are false
believes… like delusion very difficult to correct )
10. Just before the exam, you need at least 2-3 weeks for
revision (revise your mistakes , read your personal
abstracts )
Correction of false believes:
Do you have any strategy??
1. Writing summaries
2. Teaching others the new information
3. Reminding cards
4. …….
How to write your valuable summery
1. Typical presentation of the disease
2. Pathognomonic features
3. Areas of similarity and areas of difference from
other differential diagnosis (to make the question
difficult)
4. Initial investigation (cost effectiveness and sensitivity)
5. Diagnostic test (more specific)
6. Initial management (non-pharmacological + or -)
7. Most effective management (It could be costly , it
could have more side effect)
8. New update ??!!
10 Tips in answering MCQs:
1. Pay attention to the key words
2. When you chose an answer make sure the stem of the MCQ
and the answer you have chosen are grammatically correct
If you are not sure of the answer make intelligent guising:
3. Find your correct answer by process of exclusion:
 First delete the apparently incorrect answer
 Any answer partially incorrect delete it from your choices
 If two choices are similar both are incorrect
 If two choices are opposite to each other , one of them is the
correct answer
 If one choice is different from the rest, consider it the right
answer
10 Tips in answering MCQs:
4. Some MCQs ask about number . Learn how to deal with
numbers
 Arrange the answers in order – lowest->> highest
 Eliminate the lowest and highest number
 Go with the general theme of family medicine practice
 Chose the meaningful number
5. In general, respect your logical thinking & your clinical
experience
10 Tips in answering MCQs:
Examples logical thinking:
 Smoking is a risk factors of nearly any health problem
 Old medications have more side effects than new medications
 In general drug with less side effect are with weaker
effect, and the vise versa
 To change to any new intervention
(drug, procedure, investigation or vaccine …) the effect of the
new intervention must be the same or better effects and with
less side effects
10 Tips in answering MCQs:
6. In controversial issues, respect what you can remember
from your MCQs reference books
7. Don’t postponed writing in the answer sheet till near the
end of the exam time
8. Don’t hand over your paper early . Use your time fully
9. Near the end of exam time, you need 5-10 minutes for
revision to avoid stupid mistake
10. Do not change any answer unless you are 100% sure that
your first choice was wrong
A 6-month-old infant is brought into your office by his
mother. She presents with 2-day history of wheezing, a mild
fever (38.5°C), and rhinorrhea. The child has no known
allergies but there is a history of allergies in the family. On
examination, the child’s respiratory rate is 60/minute. There is
rhonchi and moist rales heard throughout the chest. The
chest x-ray shows evidence of hyperaeration.
The most likely diagnosis in the child is:
A. Mycoplasma pneumoniae infection
B. Allergic bronchitis
C. Viral tracheitis
D. Bacterial tracheitis
E. Bronchiolitis
.1
Key words:
A 6-month-old infant is brought into your office by
his mother. She presents with 2-day history of
wheezing, a mild fever (38.5°C), and rhinorrhea. The
child has no known allergies but there is a history of
allergies in the family. On examination, the child’s
respiratory rate is 60/minute. There is rhonchi and
moist rales heard throughout the chest. The chest x-
ray shows evidence of hyperaeration.
A 6-month-old infant is brought into your office by his
mother. She presents with 2-day history of wheezing, a mild
fever (38.5°C), and rhinorrhea. The child has no known
allergies but there is a history of allergies in the family. On
examination, the child’s respiratory rate is 60/minute. There is
rhonchi and moist rales heard throughout the chest. The
chest x-ray shows evidence of hyperaeration.
The most likely diagnosis in the child is:
A. Mycoplasma pneumoniae infection
B. Allergic bronchitis
C. Viral tracheitis
D. Bacterial tracheitis
E. Bronchiolitis
 Bronchiolitis (RSV) In most “typical” kids, this virus is what most parents
consider “just a cold,” but it has the potential to become serious, so here’s
what you should know
 Symptoms: Runny nose, fever, cough, wheezing with
breathing, irritability, & loss of appetite.
 Treatment
 Mild symptoms are treated with rest, fluids, and a cool air humidifier.
 Babies who are struggling to breath may hospitalized and given
supplemental humidified oxygen. Their breathing will be monitored and if
necessary fluids will be given intravenously to prevent dehydration.
 Occasionally infants need mechanical ventilation to fill and empty the lungs
until the airways open.
1. Example of Quick Revision
Bronchiolitis in infants
2. Example of Quicker Revision
1. Infants
2. Wheezing
3. Rhonchi and moist rales
4. Respiratory syncytial virus
5. Cold, humidified, oxygen
3. Example of Abstracts
Bronchiolitis in infants
A 5-year-old child is brought to the ER with his
mother. The mother tells you that for the past 24
hours the child bas been “talking strangely” and
drooling. He has had no appetite and has not been
drinking. Based on this history, what is the
diagnosis of major concern?
A. Viral pneumonia
B. Acute epiglottis
C. Bronchiolitis
D. Croup
E. Bacterial pneumonia
.2
A 5-year-old child is brought to the ER with his
mother. The mother tells you that for the past 24
hours the child bas been “talking strangely” and
drooling. He has had no appetite and has not been
drinking. Based on this history, what is the
diagnosis of major concern?
A. Viral pneumonia
B. Acute epiglottis
C. Bronchiolitis
D. Croup
E. Bacterial pneumonia
1. Example of Quick Revision
2. Example of Quicker Revision
Epiglottitis:
1. ER
2. Delusion “talking strangely”
3. Drooling
4. A lateral x-ray of the neck
5. IV/IM ceftriaxone
Epiglottitis:
3. Example of Abstracts
An 18-month-old infant is brought to the ER by his
mother. He developed an upper respirator tract
infection 2 days ago and suddenly this evening
developed a harsh, barky cough and difficulty
breathing. On examination the child is coughing. His
respiratory rate is 40/minute and he is in some
respiratory distress. The breath sounds that are
heard appear to be transmitted from the upper
airway. There are nasal flaring and
suprasternal, infrasternal, and intercostals
retractions. The child’s temperature is 38.5°C.
What is the most likely diagnosis in this child?
A. Viral pneumonia
B. Acute epiglottis
C. Bronchiolitis
D. Croup
E. Bacterial pneumonia
.3
What is the most likely diagnosis in this child?
A. Viral pneumonia - LRTI
B. Acute epiglottis - LRTI
C. Bronchiolitis - LRTI
D. Croup - URTI
E. Bacterial pneumonia - LRTI
Acute Larynotracheitis (Viral Croup)
Differential Diagnosis:
The differential diagnosis of viral croup includes
epiglottitis, foreign body, and angioneurotic edema.
Symptoms: Barking cough – especially at night; high pitched
whistling noise with inhalation (stridor); runny nose; fever.
Management:
Most children with croup do not require hospitalization. Treatment at home consists of air
humidification, avoidance of agitation, and reduction of fever
If signs of severe obstruction develop, treatment with
an aerosol of racemic epinephrine (2.25%), nebulized with 100% oxygen, frequently
provides relief. Frequent aerosol treatments may be needed for the first few hours. A single
parenteral dose of dexamethasone, 0.6 mg/kg, is effective in decreasing the length and
severity of respiratory symptoms that are associated with viral croup. Inhaled corticosteroid
therapy has also been used with some success, but parenteral dexamethasone is somewhat
easier. Intubation or tracheostomy rarely is necessary.
1. Example of Quick Revision
A systematic review of clinical
trials confirmed the observations
that nebulised adrenaline
produces immediate and often
dramatic improvements in
reducing the signs of airway
obstruction in severe croup.
Acute Larynotracheitis (Viral Croup)
2. Update & EBM
1. Early childhood
2. ER
3. Harsh, barky cough
4. Respiratory distress
5. Parainfluenza virus
6. Aerosolized steroids
7. Nebulised adrenaline
Acute Larynotracheitis (Viral Croup)
3. Example of Abstracts
Tips in studying MCQs:
 Answering one MCQ is an opportunity to review lots
of knowledge . Make sure to get the maximum from
every single MCQ . Read the explanation and review
the related topic
 Write a short abstract about each important topic
A. Write pathognomonic features
B. Write Update information
C. Write controversial issues AND stick to your NOTES
 To master clear differentiation make comparisons
between similar issues
A 25-year-old college student presents with a 3-week history
of fatigue, malaise, fever, chills, and sore throat. On physical
examination, the patient has a temperature of 39°C. There is
pharyngeal hyperemia and edema and marked exudates are
seen in both tonsillar areas. There is significant cervical
lymphadenopathy present.
You suspect infectious mononucleosis.
Of the following clinical features of acute infectious
mononucleosis, the least common is:
A. Splenomegaly
B. Hepatomegaly
C. Fever
D. Exudative tonsillitis
E. Generalized lymphadenopathy
.4
Key words
A 25-year-old college student presents with a 3-week history of
fatigue, malaise, fever, chills, and sore throat.
On physical examination, the patient has a temperature of 39°C.
There is pharyngeal hyperemia and edema and marked exudates
are seen in both tonsillar areas. There is significant cervical
lymphadenopathy present.
You suspect infectious mononucleosis.
Of the following clinical features of acute infectious
mononucleosis, the least common is:
A. Splenomegaly – (30-45%)
B. Hepatomegaly – (Impaired LFTs --30% of the patients)
C. Fever - 85%
D. Exudative tonsillitis – 60%
E. Generalized lymphadenopathy – 85%
Breast Cancer
A 41-year-old woman comes to your office after finding a breast lump
during a routine self-examination. She has been examining her breasts
regularly for the past 5 years; this is the first lump she has found. On
examination, there is a lump located in the right breast. The lump’s
anatomic location is in the upper outer quadrant. It is approximately 3
cm in diameter and is not fixed to skin or muscle. It has a hard
consistency. There are three axillary nodes present on the right side;
each node is approximately 1 cm in diameter. No lymph nodes are
present on the left. At this time, what would you do?
A. tell the patient that she has fibrocystic breast disease; ask her to
return in 1 month, preferably 10 days after the next period, for a
recheck
B tell the patient to come back for a breast examination in 6 months
C. order mammogram
D. order an ultrasound examination of the area
E. arrange for urgent fine-needle biopsy
.5
Breast Cancer
A 41-year-old woman comes to your office after finding a breast lump
during a routine self-examination. She has been examining her breasts
regularly for the past 5 years; this is the first lump she has found. On
examination, there is a lump located in the right breast. The lump’s
anatomic location is in the upper outer quadrant. It is approximately 3
cm in diameter and is not fixed to skin or muscle. It has a hard
consistency. There are three axillary nodes present on the right side;
each node is approximately 1 cm in diameter. No lymph nodes are
present on the left. At this time, what would you do?
A. tell the patient that she has fibrocystic breast disease; ask her to
return in 1 month, preferably 10 days after the next period, for a
recheck
B tell the patient to come back for a breast examination in 6 months
C. order mammogram
D. order an ultrasound examination of the area
E. arrange for urgent fine-needle biopsy
What do we mean by being specific?!
General information X Detailed information
What do we mean by being specific?!
General information X Detailed information
This is
general information
What do we mean by being specific?!
General information X Detailed information
This is
detailed information
Tips in studying MCQs:
 Be specific in your information
 And be extra specific and meticulous
Immune-deficient Child
Which one of the following vaccine must not be given to
a household contact of an immune-deficient child?
A. Mumps, measles and rubella.
B. BCG
C. Influenza vaccine
D. Oral polio vaccine
E. Hepatitis B vaccine
.6
Immune-deficient Child
Which one of the following vaccine must not be given to
a household contact of an immune-deficient child?
A. Mumps, measles and rubella.
B. BCG
C. Influenza vaccine
D. Oral polio vaccine
E. Hepatitis B vaccine
Breast Feeding
Which of the following statements about breast-feeding is
true?
A) The infant should feed on each side for 8 to 15 minutes
every 2 to 3 hours after birth.
B) Colostrum is excreted 7 to 10 days after delivery and
contains important antibodies, high calories, and other
nutrients.
C) The mother should weigh infants before and after
feeding to quantify the amount consumed.
D) Breast-feeding usually provides adequate nutrition for 2
to 4 months—supplementation should begin at that point.
E) Breast-feeding should be based on timed intervals rather
than on demand.
.7
Breast Feeding
Which of the following statements about breast-feeding is true?
A) The infant should feed on each side for 8 to 15 minutes
every 2 to 3 hours after birth.
B) Colostrum is excreted 7 to 10 days after delivery and contains
important antibodies, high calories, and other nutrients.
C) The mother should weigh infants before and after feeding to
quantify the amount consumed.
D) Breast-feeding usually provides adequate nutrition for 2 to 4
months—supplementation should begin at that point.
E) Breast-feeding should be based on timed intervals rather than
on demand.
The answer is A. Breast-feeding is encouraged for all mothers .
Currently, as many as 50% of mothers.
Growth & Development
An 8-month-old infant is brought to the emergency department by his
mother for an assessment of an upper respiratory tract infection. He has
been coughing for the past 3 days and has had a runny nose. On
examination, his temperature is 37.5°C. His weight is below the 3rd
percentile for his age, his length is at the 25th percentile, and his head
circumference is at the 50th percentile. He appears malnourished and
has thin extremities, a narrow face, prominent ribs, and wasted buttocks.
He has a prominent diaper rash, unwashed skin, a skin rash that
resembles the skin infection impetigo contagious on his face, uncut
fingernails, and dirty clothing.
 What is the most likely cause of this child’s condition?
 a. maternal deprivation
 b. cystic fibrosis
 c. constitutionally small for age
 d. infantile autism
 e. congenital bilateral sensorineural hearing loss
.8
Growth & Development
An 8-month-old infant is brought to the emergency department by his
mother for an assessment of an upper respiratory tract infection. He has
been coughing for the past 3 days and has had a runny nose. On
examination, his temperature is 37.5°C. His weight is below the 3rd
percentile for his age, his length is at the 25th percentile, and his head
circumference is at the 50th percentile. He appears malnourished and
has thin extremities, a narrow face, prominent ribs, and wasted buttocks.
He has a prominent diaper rash, unwashed skin, a skin rash that
resembles the skin infection impetigo contagiosum on his face, uncut
fingernails, and dirty clothing.
 What is the most likely cause of this child’s condition?
 a. maternal deprivation
 b. cystic fibrosis
 c. constitutionally small for age
 d. infantile autism
 e. congenital bilateral sensorineural hearing loss
Growth & Development
Bone age can sometimes be used to differentiate certain
causes of short stature in children. With respect to bone
age, which of the following statements is true?
A. bone age is normal in both familial short stature and
constitutional delay of growth
B. bone age is normal in familial short stature and delayed
in constitutional delay of growth
C. bone age is normal in constitutional delay of growth and
delayed in growth hormone deficiency
D. bone age is delayed in both familial short stature and
short stature caused by hypothyroidism
E. bone age is variable and cannot be used to differentiate
familial short stature and constitutional delay
.9
Growth & Development
Bone age can sometimes be used to differentiate certain
causes of short stature in children. With respect to bone
age, which of the following statements is true?
A. bone age is normal in both familial short stature and
constitutional delay of growth
B. bone age is normal in familial short stature and delayed
in constitutional delay of growth
C. bone age is normal in constitutional delay of growth and
delayed in growth hormone deficiency
D. bone age is delayed in both familial short stature and
short stature caused by hypothyroidism
E. bone age is variable and cannot be used to differentiate
familial short stature and constitutional delay
Teeth Eruption
A 9-month-old boy brought by his mother to see you in
the clinic. Her main concern that her son did not have
teeth eruption till now. He is developmentally normal. His
height and weight are normal. What will be your advice
to her?
A. Tell her don't worry except after 1 month.
B. It is not unusual for the first deciduous teeth to erupt
before 12 months.
C. Investigate for hypothyroidism.
D. Do roentgenogram for mandible and maxillary X ray.
E. Tell her that because her son have a nails, so he will also
have teeth eruption.
.10
Teeth Eruption
A 9-month-old boy brought by his mother to see you in
the clinic. Her main concern that her son did not have
teeth eruption till now. He is developmentally normal. His
height and weight are normal. What will be your advice
to her?
A. Tell her don't worry except after 1 month.
B. It is not unusual for the first deciduous teeth to erupt
before 12 months.
C. Investigate for hypothyroidism.
D. Do roentgenogram for mandible and maxillary X ray.
E. Tell her that because her son have a nails, so he will also
have teeth eruption.
Visual Screening
Which statement regarding visual screening in children is
correct?
A. Visual screening is not indicated until age 5.
B. Visual acuity can be assessed by the Random Dot E test.
C. Stereopsis can be measured by the Tumbling E test.
D. Strabismus can be assessed with the cover test during
the first year of life.
E. Visual screening can be reliably assessed at age 2.
.11
Visual Screening
Which statement regarding visual screening in children is
correct?
A. Visual screening is not indicated until age 5.
B. Visual acuity can be assessed by the Random Dot E test.
C. Stereopsis can be measured by the Tumbling E test.
D. Strabismus can be assessed with the cover test during
the first year of life.
E. Visual screening can be reliably assessed at age 2.
Influenza Vaccination
Which one of the following patients should not
receive an influenza vaccination?
A. A 36-year-old pregnant woman at 12 weeks'
gestation.
B. A 65-year-old man with diabetes mellitus.
C. A 28-year-old man with HIV infection.
D. A 54-year-old woman who is allergic to eggs.
E. A six-year-old girl who lives with a grandmother
who has terminal cancer.
.12
Influenza Vaccination
Which one of the following patients should not receive
an influenza vaccination?
A. A 36-year-old pregnant woman at 12 weeks' gestation.
B. A 65-year-old man with diabetes mellitus.
C. A 28-year-old man with HIV infection.
D. A 54-year-old woman who is allergic to eggs.
E. A six-year-old girl who lives with a grandmother who
has terminal cancer.
Chose the different
Knowledge of Vaccination
A 68-year-old patient is seen for a general examination.
Current recommendations for immunizations include
A. tetanus booster every 5 years
B. influenza vaccination yearly
C. pneumococcal vaccination yearly
D. hepatitis booster every 5 years
E. meningococcal vaccination
.13
Knowledge of Vaccination
A 68-year-old patient is seen for a general examination.
Current recommendations for immunizations include
A. tetanus booster every 5 years
B. influenza vaccination yearly
C. pneumococcal vaccination yearly
D. hepatitis booster every 5 years
E. meningococcal vaccination
Tips in studying MCQs:
 Concentrate on important topics in family
medicine e.g.:
 Children growth and development
 Breast feeding
 Vaccination
 Presentation of common diseases special patients. e.g.
appendicitis in very young and in very elderly patients
You will be asked in this topics in any FM exam
Forgetfulness
A 57-year-old male has become
forgetful, preoccupied, withdrawn and suspicious. His
physical examination was normal. The patient has
been with his company for 22 years and was
considered an excellent employee. Which of the
following is most likely diagnosis?
A. Multi-infarct dementia.
B. Depression.
C. Vascular dementia
D. Alcoholism.
E. Alzheimer
.14
Forgetfulness
A 57-year-old male has become
forgetful, preoccupied, withdrawn and suspicious. His
physical examination was normal. The patient has
been with his company for 22 years and was
considered an excellent employee. Which of the
following is most likely diagnosis?
A. Multi-infarct dementia.
B. Depression.
C. Vascular dementia
D. Alcoholism.
E. Alzheimer
Tips in answering MCQs:
If you are not sure of the answer make inelegant guising:
Find your correct answer by process of exclusion:
 First delete the apparently incorrect newer
 If two choices are similar both are incorrect
Post MI Medications
Which of the following improve survival after myocardial
infarction in hypertensive patient?
A. Nitrate.
B. Morphine.
C. ACE inhibitors.
D. Calcium channel blockers.
E. Aspirin.
.15
Post MI Medications
Which of the following improve survival after myocardial
infarction in hypertensive patient?
A. Nitrate.
B. Morphine.
C. ACE inhibitors.
D. Calcium channel blockers.
E. Aspirin.
H. Pylori Testing
.16
Which of the following types of H. pylori testing is not
useful to confirm eradication?
A. stool antigen test
B. urea breath test
C. enzyme-linked immunosorbent assay (ELISA)
D. Serology culture
E. Steiner stain of gastric biopsy specimen
H. Pylori Testing
Which of the following types of H. pylori testing is not
useful to confirm eradication?
A. stool antigen test
B. urea breath test
C. enzyme-linked immunosorbent assay (ELISA)
D. Serology culture
E. Steiner stain of gastric biopsy specimen
Gout
Which one of these drug is not associated with gout?
A. Diuretics
B. Digoxin
C. Niacin (vitamin B3)
D. Aspirin
E. Cyclosporine (Immunosuppressive)
.17
Gout
Which one of these drug is not associated with gout?
A. Diuretics
B. Digoxin
C. Niacin (vitamin B3)
D. Aspirin
E. Cyclosporine (Immunosuppressive)
Vague Chest Pain
 A 35-year-old male consults you about vague chest pain he
developed while sitting at his desk earlier in the day. The
pain is right-sided and was sharp for a brief time when it
began, but it rapidly subsided. There was no hemoptysis
and the pain does not seem pleuritic. His physical
examination, EKG, and oxygen saturation are
unremarkable. A chest film shows a 10% right
pneumothorax. Which one of the following should you do
next?
A. Admit the patient to the hospital for observation.
B. Admit the patient to the hospital for chest tube
placement.
C. Obtain a repeat chest radiograph in 24–48 hours.
D. Obtain an expiratory chest radiograph.
E. Reassure the patient and do nothing
.18
Vague Chest Pain
 A 35-year-old male consults you about vague chest pain he
developed while sitting at his desk earlier in the day. The
pain is right-sided and was sharp for a brief time when it
began, but it rapidly subsided. There was no hemoptysis
and the pain does not seem pleuritic. His physical
examination, EKG, and oxygen saturation are
unremarkable. A chest film shows a 10% right
pneumothorax. Which one of the following should you do
next?
A. Admit the patient to the hospital for observation.
B. Admit the patient to the hospital for chest tube
placement.
C. Obtain a repeat chest radiograph in 24–48 hours.
D. Obtain an expiratory chest radiograph.
E. Reassure the patient and do nothing
Fatigue and Weakness
A 60-year-old man is being evaluated for fatigue, weakness, and
exercise intolerance. Laboratory assessment reveals:
Hemoglobin: 9.1 mg/dL (L)
Serum iron: 46 μg/dL (L)
Ferritin: 9 ng/mL (L)
Total iron binding capacity (TIBC): 626 μg/dL (H)
Mean corpuscular volume (MCV): 76 fL (L)
What is the most common cause of this condition?
A. Blood loss
B. Poor nutrition
C. Inadequate absorption of iron
D. Chronic disease
E. Folic acid deficiency
.19
Fatigue and Weakness
A 60-year-old man is being evaluated for fatigue, weakness, and
exercise intolerance. Laboratory assessment reveals:
Hemoglobin: 9.1 mg/dL (L)
Serum iron: 46 μg/dL (L)
Ferritin: 9 ng/mL (L)
Total iron binding capacity (TIBC): 626 μg/dL (H)
Mean corpuscular volume (MCV): 76 fL (L)
What is the most common cause of this condition?
A. Blood loss
B. b. Poor nutrition
C. c. Inadequate absorption of iron
D. d. Chronic disease
E. e. Folic acid deficiency
Liver Disease with Cirrhosis
 You are seeing a patient who has end-stage liver
disease with cirrhosis due to hepatitis C. What will be
her most likely cause of death?
A. Liver failure.
B. Myocardial infarction.
C. Bleeding varices.
D. Hepatocellular carcinoma.
E. Renal failure.
.20
Liver Disease with Cirrhosis
 You are seeing a patient who has end-stage liver
disease with cirrhosis due to hepatitis C. What will be
her most likely cause of death?
A. Liver failure.
B. Myocardial infarction.
C. Bleeding varices.
D. Hepatocellular carcinoma.
E. Renal failure.
The main causes of 436 deaths among 532 patients with cirrhosis followed up for up to 16
years constituted liver failure (24%), liver failure with gastrointestinal bleeding
(13%), gastrointestinal bleeding (14%) i.e. bleeding ±hepatic failure (27%), primary liver cell
carcinoma (4%), other liver-related causes (2%), infections (7%), cardiovascular diseases
(22%), extra hepatic malignancies (9%), and other non-liver-related causes (5%). Totally, 57%
died of liver-related causes
.20
Liver Disease with Cirrhosis
 You are seeing a patient who has end-stage liver
disease with cirrhosis due to hepatitis C. What will be
her most likely cause of death?
A. Liver failure.
B. Myocardial infarction.
C. Bleeding varices.
D. Hepatocellular carcinoma.
E. Renal failure.
The main causes of 436 deaths among 532 patients with cirrhosis followed up for up to 16
years constituted liver failure (24%), liver failure with gastrointestinal bleeding
(13%), gastrointestinal bleeding (14%) i.e. bleeding ±hepatic failure (27%), primary liver cell
carcinoma (4%), other liver-related causes (2%), infections (7%), cardiovascular diseases
(22%), extra hepatic malignancies (9%), and other non-liver-related causes (5%). Totally, 57%
died of liver-related causes
Prostate Cancer
The best management of localized, well-differentiated
prostate cancer in men older than 65 is:
A. Radiation implants.
B. External beam radiation therapy.
C. Watchful waiting.
D. Primary androgen deprivation therapy.
E. Robot-assisted prostatectomy.
.21
Prostate Cancer
The best management of localized, well-differentiated
prostate cancer in men older than 65 is:
A. Radiation implants.
B. External beam radiation therapy.
C. Watchful waiting.
D. Primary androgen deprivation therapy.
E. Robot-assisted prostatectomy.
Tips in studying MCQs:
 Be specific in your information
 And be extra specific and meticulous
AAA
Which one of the following is the greatest risk factor
for abdominal aortic aneurysm (AAA)?
A. Cigarette smoking.
B. Diabetes mellitus.
C. Hypertension.
D. African-American race.
E. Female gender.
.22
AAA
Which one of the following is the greatest risk factor
for abdominal aortic aneurysm (AAA)?
A. Cigarette smoking.
B. Diabetes mellitus.
C. Hypertension.
D. African-American race.
E. Female gender.
Abdominal Aortic Aneurysm (AAA)
 In the UK the rate of Abdominal Aortic Aneurism (AAA) in
Caucasian men older than 65 years is about 4.7%
 The peak incidence is among males around 70 years of
age, the prevalence among males over 60 years totals 2-
6%.
 In the U.S. the incidence of AAA is 2-4% in the adult
population.
 AAA is 4-6 times more common in male siblings of known
patients, with a risk of 20-30%.
 Rupture of the AAA occurs in 1-3% of men aged 65 or
more, the mortality is 70-95%.
Smoking & AAA
Cervical Cancer
Which of the following statements regarding cervical
cancer screening is true?
A. The death rate from cervical cancer continues to
increase despite Pap smear screening.
B. Immunosuppression has not been identified as a risk
factor for cervical cancer.
C. Human papillomavirus (HPV) types 1 and 3 are most
closely linked to cervical cancer.
D. Smoking has been linked to the development of cervical
cancer.
E. Most cases of cervical cancer occur between the ages of
20 and 30 years.
.23
Cervical Cancer
Which of the following statements regarding cervical cancer
screening is true?
A. The death rate from cervical cancer continues to increase despite
Pap smear screening.
B. Immunosuppression has not been identified as a risk factor for
cervical cancer.
C. Human papillomavirus (HPV) types 1 and 3 are most closely linked
to cervical cancer.
D. Smoking has been linked to the development of cervical cancer.
E. Most cases of cervical cancer occur between the ages of 20 and 30
years.
 The answer is D.
 Tip: Smoking always the first to blame
Tips in answering MCQs:
Examples logical thinking:
Smoking is the first to blame …
Smoking is “ criminal ”
Smoking Risks
 In the British Journal of Psychiatry, the researchers wrote, “The
best-fitting causal model was one in which nicotine
dependence led to increased risk of depression.” They suggest
two possible routes, one involving common risk factors, and the
second a direct causal link.
 According to the researchers, “this evidence is consistent with
the conclusion that there is a cause and effect relationship
between smoking and depression in which cigarette smoking
increases the risk of symptoms of depression.”
 http://psychcentral.com/lib/can-smoking-cause-depression/0007153
Smoking and Depression
Smoking
Which of the following conditions is NOT associated with
smoking?
A. Peptic ulcer disease
B. Depression
C. Children of smokers are at increased risk for otitis media
D. Osteoporosis
E. Cervical cancer
.24
Smoking
Which of the following conditions is NOT associated with
smoking?
A. Peptic ulcer disease
B. Depression
C. Children of smokers are at increased risk for otitis media
D. Osteoporosis
E. Cervical cancer
Tips in answering MCQs:
 In general, respect your clinical experience
 In controversial issues, respect what you can
remember from your MCQs reference books
Hepatitis C & Brest Milk
Which of the following statements is correct concerning
hepatitis C virus (HCV)?
A. There is no risk to infants if the mother is affected.
B. There is no risk associated with sexual intercourse with
an individual with hepatitis C.
C. Cesarean section should be performed on mothers who
test positive for hepatitis C to prevent transmission to
the newborn.
D. Hepatitis C can be spread by contaminated water
supplies.
E. Hepatitis C does not appear to be transmitted in breast
milk.
.25
Hepatitis C & Brest Milk
Which of the following statements is correct concerning
hepatitis C virus (HCV)?
A. There is no risk to infants if the mother is affected.
B. There is no risk associated with sexual intercourse with
an individual with hepatitis C.
C. Cesarean section should be performed on mothers who
test positive for hepatitis C to prevent transmission to
the newborn.
D. Hepatitis C can be spread by contaminated water
supplies.
E. Hepatitis C does not appear to be transmitted in breast
milk.
1. Learn to write very clear message to correct any false believes
in your MCQs Knowledge
2. It is more helpful if you add new relevant information to
maximize your benefit from MCQs studying
Tips in studying MCQs:
 Answering one MCQ is an opportunity to review lots of
knowledge . Make sure to get the maximum from every
single MCQ . Read the explanation and review the related
topic
 Write a short abstract about each important topic
Headache Red Flags
Each of the following is a solid indication for
neuroimaging in a patient with headache except:
A. Onset of headaches over the age of 50 years
B. Seizures associated
C. Prolonged aura
D. Nausea and vomiting
E. Headache worsening with movement
.26
Headache Red Flags
Each of the following is a solid indication for
neuroimaging in a patient with headache except:
A. Onset of headaches over the age of 50 years
B. Seizures associated
C. Prolonged aura
D. Nausea and vomiting
E. Headache worsening with movement
Head Lice
Which of the following is true regarding management of
head lice?
A. Children should be kept out of school until no visible
evidence of nits is noted.
B. Household members should only be treated if live lice or
eggs are noted within 1 cm of the scalp.
C. Head lice programs have had a significant impact on
lowering the incidence of head lice.
D. Cleaning of bedding has little impact on lice eradication.
E. The health of those exposed is more important than the
confidentiality of the child affected.
.27
Head Lice
Which of the following is true regarding management of head lice?
A. Children should be kept out of school until no visible evidence of
nits is noted.
B. Household members should only be treated if live lice or eggs are
noted within 1 cm of the scalp.
C. Head lice programs have had a significant impact on lowering the
incidence of head lice.
D. Cleaning of bedding has little impact on lice eradication.
E. The health of those exposed is more important than the
confidentiality of the child affected.
The answer is B. Practice guidelines published by the American
Academy of Pediatrics (AAP) state that if a case of head lice is
identified, all household members should be checked, and only
those with live lice or eggs within 1 cm of the scalp should be
treated. It is recommended to treat family members who share a
bed with the person who is infected
Tips in answering MCQs:
 In general, respect your clinical experience
 If your clinical practice is different than MCQ correct
answer according to the reference book . Go with the
book
Number Technique
Before the diagnosis of post-traumatic stress disorder
(PTSD) is made, symptoms should be present for at
least
A. 1 year
B. 6 months
C. 3 months
D. 1 month
E. 1 week
.28
Number Technique
Before the diagnosis of post-traumatic stress disorder
(PTSD) is made, symptoms should be present for at
least
A. 1 year
B. 6 months
C. 3 months
D. 1 month
E. 1 week
Number /Anorexia Nervosa
In young women with an eating disorder, at what
point would you expect her menstrual periods to
resume?
A. 75% of ideal body weight
B. 80% of ideal body weight
C. 90% of ideal body weight
D. 100% of body weight
E. It is unusual for menstrual cycles to resume with
any weight gain.
.29
Number /Anorexia Nervosa
In young women with an eating disorder, at what
point would you expect her menstrual periods to
resume?
A. 75% of ideal body weight
B. 80% of ideal body weight
C. 90% of ideal body weight
D. 100% of body weight
E. It is unusual for menstrual cycles to resume with
any weight gain.
In a healthy patient, what is the appropriate age to
administer Pneumococcal vaccine?
A. 50 years
B. 5 years
C. 75 years
D. 16 years
E. 65 years
.30
In a healthy patient, what is the appropriate age to
administer Pneumococcal vaccine?
A. 5 years
B. 16 years
C. 50 years
D. 65 years
E. 75 years
Tips in answering MCQs:
Some MCQs ask about number . Learn how to deal with
numbers
 Arrange the answers in order – lowest->> highest
 Eliminate the lowest and highest number
 Go with the general theme of family medicine practice
 Chose the meaningful number
Use of Antibiotics
Which one of the following is appropriate and
effective treatment for genitourinary gonorrhea in a
20-year-old male with a purulent urethral discharge?
A. Amoxicillin, 3.5 g orally once.
B. Ciprofloxacin, 500 mg orally once.
C. Ceftriaxone, 125 mg intramuscularly once.
D. Doxycycline, 100 mg 2 times daily for 3 days.
E. Erythromycin, 500 mg 4 times daily for 7 days.
.31
Use of Antibiotics
Which one of the following is appropriate and
effective treatment for genitourinary gonorrhea in a
20-year-old male with a purulent urethral discharge?
A. Amoxicillin, 3.5 g orally once.
B. Ciprofloxacin, 500 mg orally once.
C. Ceftriaxone, 125 mg intramuscularly once.
D. Doxycycline, 100 mg 2 times daily for 3 days.
E. Erythromycin, 500 mg 4 times daily for 7 days.
Use of Antibiotics
Of the following antibiotics, which one would be
acceptable to use when treating penicillin-resistant S.
pneumoniae otitis media?
A. Azithromycin
B. Clarithromycin
C. Cefuroxime
D. Cefaclor
E. Cephalexin
.32
Use of Antibiotics
Of the following antibiotics, which one would be
acceptable to use when treating penicillin-resistant S.
pneumoniae otitis media?
A. Azithromycin
B. Clarithromycin
C. Cefuroxime
D. Cefaclor
E. Cephalexin
Only five antibiotics—high-dose amoxicillin (80
mg/kg/day), amoxicillin–clavulanate (Augmentin), cefuroxime
(Ceftin), cefprozil (Cefzil), and ceftriaxone (Rocephin)—have
demonstrated a modest degree (60% to 80%) of clinical efficacy in
the treatment of acute otitis media caused by penicillin-resistant
S. pneumoniae.
Antibiotics
An 18-year-old woman presents with
swelling, warmth, and spreading redness at the upper
part of her ear, where she recently underwent an ear
piercing. Appropriate antibiotic coverage includes
A. cephalexin
B. ciprofloxacin
C. azithromycin
D. penicillin
E. tetracycline
.33
Antibiotics
An 18-year-old woman presents with
swelling, warmth, and spreading redness at the upper
part of her ear, where she recently underwent an ear
piercing. Appropriate antibiotic coverage includes
A. cephalexin
B. ciprofloxacin
C. azithromycin
D. penicillin
E. tetracycline
Depression
A 14-year-old girl is diagnosed with major depressive
disorder. It is her first episode and the clinical
assessment is that it is mild depression. Which one of
the following treatment choices is appropriate?
A. Tricyclic antidepressant..
B. SSRI.
C. St. John's wart.
D. Cognitive-behavioral therapy.
E. Lithium.
.34
Depression
A 14-year-old girl is diagnosed with major depressive
disorder. It is her first episode and the clinical
assessment is that it is mild depression. Which one of
the following treatment choices is appropriate?
A. Tricyclic antidepressant..
B. SSRI.
C. St. John's wart.
D. Cognitive-behavioral therapy.
E. Lithium.
Alcoholism management
 A 56-year-old business executive with a history of
alcoholism and associated liver disease presents to
your office and would like to stop drinking. In order to
prevent alcohol withdrawal, you select which of the
following medications?
A. Lorazepam
B. Clonazepam
C. Diazepam
D. Flurazepam
E. Buspirone
.35
Alcoholism management
 A 56-year-old business executive with a history of
alcoholism and associated liver disease presents to
your office and would like to stop drinking. In order to
prevent alcohol withdrawal, you select which of the
following medications?
A. Lorazepam
B. Clonazepam
C. Diazepam
D. Flurazepam
E. Buspirone
Anti depressant Medications
Which of the following medications is least likely to cause
sexual side effects?
A. Fluoxetine (Prozac)
B. Sertraline (Zoloft)
C. Venlafaxine (Effexor)
D. Citalopram (Celexa)
E. Bupropion (Wellbutrin)
.36
Anti depressant Medications
Which of the following medications is least likely to cause
sexual side effects?
A. Fluoxetine (Prozac)
B. Sertraline (Zoloft)
C.Venlafaxine (Effexor)
D. Citalopram (Celexa)
E. Bupropion (Wellbutrin)
Antidepressant
About how many patients in the total treated
population respond only partially or don't respond
at all to antidepressant monotherapy?
A. 5 to 20 percent.
B. 10 to 30 percent.
C. 20 to 40 percent.
D. 30 to 50 percent.
E. 40 to 60 percent.
.37
Antidepressant
About how many patients in the total treated
population respond only partially or don't respond
at all to antidepressant monotherapy?
A. 5 to 20 percent.
B. 10 to 30 percent.
C. 20 to 40 percent.
D. 30 to 50 percent.
E. 40 to 60 percent.
Placebo effect is up to 60%
About how many patients in the total treated
population respond only partially or don't respond
at all to antidepressant monotherapy?
A. 5 to 20 percent.
B. 10 to 30 percent.
C. 20 to 40 percent.
D. 30 to 50 percent.
E. 40 to 60 percent.
Placebo effect is up to 60%
Antidepressant
Tips in studying MCQs:
Concentrate on important topics in family
medicine e.g.: Important Medications
• Antibiotics
• Antidepressant
• Other psychotropic medications
• Anti epilepsy medications
• Anti TB
• Anti ulcer medications
• Antihypertensive medications
• Hypoglycemic medications
• Asthma medications
Hepatitis C Infection
Which one of the following is the approximate
risk that a health care worker will contract
hepatitis C infection following a needle stick
injury contaminated by blood from a patient
testing positive for hepatitis C by polymerase
chain reaction?
A. 0.6 percent.
B. 50 percent.
C. 6.1 percent.
D. 16 percent.
E. 26 percent.
.38
Hepatitis C Infection
Which one of the following is the approximate
risk that a health care worker will contract
hepatitis C infection following a needle stick
injury contaminated by blood from a patient
testing positive for hepatitis C by polymerase
chain reaction?
A. 0.6 percent.
B. 50 percent.
C. 6.1 percent.
D. 16 percent.
E. 26 percent.
Which one of the following is the approximate
risk that a health care worker will contract
hepatitis C infection following a needle stick
injury contaminated by blood from a patient
testing positive for hepatitis C by polymerase
chain reaction?
A. 0.6 percent.
B. 50 percent.
C. 6.1 percent.
D. 16 percent.
E. 26 percent.
Hepatitis C Infection
Urinary Tract Infections
Which one of the following is the most sensitive
laboratory indicator for urinary tract infections?
A. Pyuria.
B. Bacteriuria.
C. Leukocyte esterase.
D. Increased vaginal pH.
E. Urine nitrite.
.39
Urinary Tract Infections
Which one of the following is the most sensitive
laboratory indicator for urinary tract infections?
A. Pyuria.
B. Bacteriuria.
C. Leukocyte esterase.
D. Increased vaginal pH.
E. Urine nitrite.
Respect your practical experience
HIV
In which one of the following situations should a health care
worker be offered antiretroviral prophylaxis after an
occupational exposure to a patient known to be infected
with human immunodeficiency virus?
A. Urine with no visible blood in it splashed into the worker's
eyes.
B. Needle-stick injury when recapping a clean needle.
C. Intact skin contact with patient's tears.
D. Accidental cut from a used sharp instrument.
E. Urine with no visible blood in it splashed onto exposed skin.
.40
HIV
In which one of the following situations should a health care
worker be offered antiretroviral prophylaxis after an
occupational exposure to a patient known to be infected
with human immunodeficiency virus?
A. Urine with no visible blood in it splashed into the worker's
eyes.
B. Needle-stick injury when recapping a clean needle.
C. Intact skin contact with patient's tears.
D. Accidental cut from a used sharp instrument.
E. Urine with no visible blood in it splashed onto exposed skin.
Common sense
HIV Infection
What is the approximate risk of (HIV) seroconversion
after a needle-stick injury when the source patient
has documented HIV infection?
A. 0.01 to 0.05 percent.
B. 0.3 to 0.5 percent.
C. 1.0 to 5.0 percent.
D. 25 to 30 percent.
E. Nearly 100 percent.
.41
HIV Infection
What is the approximate risk of (HIV)
seroconversion after a needle-stick injury when the
source patient has documented HIV infection?
A. 0.01 to 0.05 percent.
B. 0.3 to 0.5 percent.
C. 1.0 to 5.0 percent.
D. 25 to 30 percent.
E. Nearly 100 percent.
Needle Stick injury &
Risk to the Health Care Worker
 HIV
 Hepatitis C
 Hepatitis B
 0.3 to 0.5 percent
 6.1 percent
 ….
Tips in studying MCQs:
Concentrate on important topics in family
medicine e.g.: Important infections
 TB
 Hepatitis B & C
 HIV
 Any current epidemic disease TB
Falls In The Elderly
Which of the following is responsible for the largest
proportion of falls in the elderly?
A. Extrinsic (environmental) factors.
B. Postural changes.
C. Dizziness.
D. Gait problems.
E. Impaired vision.
.42
Falls In The Elderly
Which of the following is responsible for the largest
proportion of falls in the elderly?
A. Extrinsic (environmental) factors.
B. Postural changes.
C. Dizziness.
D. Gait problems.
E. Impaired vision.
Different
Asymptomatic Bacteriuria
Of the following outcomes, which one is associated
with antibiotic treatment of asymptomatic
bacteriuria in elderly women?
A. Prevention of pyelonephritis.
B. Prevention of hypertension.
C. Sterilization of the urinary tract.
D. Emergence of drug-resistant bacterial strains.
E. Overall reduction in mortality.
.43
Asymptomatic Bacteriuria
Of the following outcomes, which one is associated
with antibiotic treatment of asymptomatic
bacteriuria in elderly women?
A. Prevention of pyelonephritis.
B. Prevention of hypertension.
C. Sterilization of the urinary tract.
D. Emergence of drug-resistant bacterial strains.
E. Overall reduction in mortality.
Different
Tips in answering MCQs:
and the answer you have chosen are grammatically correct
If you are not sure of the answer make intelligent guising:
 If one choice is different from the rest, consider it the
right answer
Acellular Pertussis Vaccine
Which one of the following statements about the
acellular pertussis vaccine is true?
A. It is about as effective as the whole-cell vaccine with fewer
side effects.
B. It is a little less effective than the whole-cell vaccine with
fewer side effects.
C. It is as effective as the whole-cell vaccine but does not show
an appreciable difference in side effects.
D. It is less effective than the whole-cell vaccine with no
significant difference in side effects.
E. It is less effective than the whole-cell vaccine with more
side effects.
.44
Which one of the following statements about the
acellular pertussis vaccine is true?
A. It is about as effective as the whole-cell vaccine with fewer
side effects.
B. It is a little less effective than the whole-cell vaccine with
fewer side effects.
C. It is as effective as the whole-cell vaccine but does not show
an appreciable difference in side effects.
D. It is less effective than the whole-cell vaccine with no
significant difference in side effects.
E. It is less effective than the whole-cell vaccine with more
side effects.
New vaccine has same or better effects
and less side effects
Tips in answering MCQs:
Examples logical thinking:
 To change to any new intervention
(drug, procedure, investigation or vaccine …) the effect of the
new intervention must be the same or better effects and with
less side effects
Thrombocytopenic Purpura
The following statement about thrombocytopenic
purpura is correct?
A. They present most often with nosebleeds or
bleeding gums.
B. They tend to be girls.
C. They remit spontaneously in up to 90 percent of
cases.
D. They have a slightly higher risk of intracerebral
hemorrhage than adults.
E. They show poikilocytosis and nucleated red cells on
the peripheral smear.
.45
Thrombocytopenic Purpura
The following statement about thrombocytopenic
purpura is correct?
A. They present most often with nosebleeds or
bleeding gums.
B. They tend to be girls.
C. They remit spontaneously in up to 90 percent of
cases.
D. They have a slightly higher risk of intracerebral
hemorrhage than adults.
E. They show poikilocytosis and nucleated red cells on
the peripheral smear.
Tips in answering MCQs:
Respect your logical thinking & your clinical experience
Thank yo

Más contenido relacionado

La actualidad más candente

General management of trauma
General management of traumaGeneral management of trauma
General management of traumaAhmad Sulong
 
Patient Diary by Abhinandan Sandhu
Patient Diary by Abhinandan SandhuPatient Diary by Abhinandan Sandhu
Patient Diary by Abhinandan Sandhuabhigill
 
advanced trauma life support
advanced trauma life supportadvanced trauma life support
advanced trauma life supportDr. SHEETAL KAPSE
 
Breaking bad news ----
Breaking bad news ----Breaking bad news ----
Breaking bad news ----Pk Doctors
 
Internal Medicine Board Review - Neurology Flashcards - by Knowmedge
Internal Medicine Board Review - Neurology Flashcards - by KnowmedgeInternal Medicine Board Review - Neurology Flashcards - by Knowmedge
Internal Medicine Board Review - Neurology Flashcards - by KnowmedgeKnowmedge
 
Penetrating neck trauma
Penetrating neck traumaPenetrating neck trauma
Penetrating neck traumaSCGH ED CME
 
Questions 1700 - Medical MCQ without Answer
Questions 1700 - Medical MCQ without AnswerQuestions 1700 - Medical MCQ without Answer
Questions 1700 - Medical MCQ without AnswerMBBS Help
 
Protocol ppt
Protocol pptProtocol ppt
Protocol pptaishuanju
 
AETCOM module 2.6
AETCOM module 2.6AETCOM module 2.6
AETCOM module 2.6Ankita Bist
 
Introduction to Competency-based Medical Education
Introduction to Competency-based Medical EducationIntroduction to Competency-based Medical Education
Introduction to Competency-based Medical EducationImad Hassan
 
Penetrating neck injury
Penetrating neck injuryPenetrating neck injury
Penetrating neck injuryNote Noteenote
 

La actualidad más candente (20)

Doctor – Patient Communication By Dr. Ashok Balsekar
Doctor – Patient Communication By Dr. Ashok BalsekarDoctor – Patient Communication By Dr. Ashok Balsekar
Doctor – Patient Communication By Dr. Ashok Balsekar
 
General management of trauma
General management of traumaGeneral management of trauma
General management of trauma
 
Patient Diary by Abhinandan Sandhu
Patient Diary by Abhinandan SandhuPatient Diary by Abhinandan Sandhu
Patient Diary by Abhinandan Sandhu
 
Introduction To ATLS
Introduction To ATLSIntroduction To ATLS
Introduction To ATLS
 
advanced trauma life support
advanced trauma life supportadvanced trauma life support
advanced trauma life support
 
Medical ethics
Medical ethicsMedical ethics
Medical ethics
 
Breaking bad news ----
Breaking bad news ----Breaking bad news ----
Breaking bad news ----
 
Internal Medicine Board Review - Neurology Flashcards - by Knowmedge
Internal Medicine Board Review - Neurology Flashcards - by KnowmedgeInternal Medicine Board Review - Neurology Flashcards - by Knowmedge
Internal Medicine Board Review - Neurology Flashcards - by Knowmedge
 
Penetrating neck trauma
Penetrating neck traumaPenetrating neck trauma
Penetrating neck trauma
 
Questions 1700 - Medical MCQ without Answer
Questions 1700 - Medical MCQ without AnswerQuestions 1700 - Medical MCQ without Answer
Questions 1700 - Medical MCQ without Answer
 
Protocol ppt
Protocol pptProtocol ppt
Protocol ppt
 
Clinical History Taking
Clinical History TakingClinical History Taking
Clinical History Taking
 
MCQs
MCQsMCQs
MCQs
 
Polytrauma
Polytrauma Polytrauma
Polytrauma
 
Case files usmle step 1 ( sample )
Case files usmle step 1 ( sample )Case files usmle step 1 ( sample )
Case files usmle step 1 ( sample )
 
AETCOM module 2.6
AETCOM module 2.6AETCOM module 2.6
AETCOM module 2.6
 
MCQ Workshop - Dr Jane Holland
MCQ Workshop - Dr Jane HollandMCQ Workshop - Dr Jane Holland
MCQ Workshop - Dr Jane Holland
 
Introduction to Competency-based Medical Education
Introduction to Competency-based Medical EducationIntroduction to Competency-based Medical Education
Introduction to Competency-based Medical Education
 
Penetrating neck injury
Penetrating neck injuryPenetrating neck injury
Penetrating neck injury
 
Informed Consent powerpoint
Informed Consent powerpointInformed Consent powerpoint
Informed Consent powerpoint
 

Destacado

Vaginal discharge & pruritis vulvae
Vaginal discharge & pruritis vulvaeVaginal discharge & pruritis vulvae
Vaginal discharge & pruritis vulvaemagdy abdel
 
Chapter 3 presentation
Chapter 3 presentationChapter 3 presentation
Chapter 3 presentationPatty Ramirez
 
Pruritus Vulva - www.jinekolojivegebelik.com
Pruritus Vulva - www.jinekolojivegebelik.comPruritus Vulva - www.jinekolojivegebelik.com
Pruritus Vulva - www.jinekolojivegebelik.comjinekolojivegebelik.com
 
Consultation models
Consultation modelsConsultation models
Consultation modelsFayzaRayes
 
Dermatology approach
Dermatology approachDermatology approach
Dermatology approachFayzaRayes
 
Pregnancy Dermatoses
Pregnancy Dermatoses Pregnancy Dermatoses
Pregnancy Dermatoses Ibrahim Farag
 
Skin changes and dermatoses of pregnancy
Skin changes and dermatoses of pregnancySkin changes and dermatoses of pregnancy
Skin changes and dermatoses of pregnancyMEEQAT HOSPITAL
 

Destacado (10)

Skin Examination
Skin ExaminationSkin Examination
Skin Examination
 
Vaginal discharge & pruritis vulvae
Vaginal discharge & pruritis vulvaeVaginal discharge & pruritis vulvae
Vaginal discharge & pruritis vulvae
 
Pruritis Ani
Pruritis AniPruritis Ani
Pruritis Ani
 
Chapter 3 presentation
Chapter 3 presentationChapter 3 presentation
Chapter 3 presentation
 
Pruritus Vulva - www.jinekolojivegebelik.com
Pruritus Vulva - www.jinekolojivegebelik.comPruritus Vulva - www.jinekolojivegebelik.com
Pruritus Vulva - www.jinekolojivegebelik.com
 
Consultation models
Consultation modelsConsultation models
Consultation models
 
Dermatology approach
Dermatology approachDermatology approach
Dermatology approach
 
Pregnancy Dermatoses
Pregnancy Dermatoses Pregnancy Dermatoses
Pregnancy Dermatoses
 
Pruritus
PruritusPruritus
Pruritus
 
Skin changes and dermatoses of pregnancy
Skin changes and dermatoses of pregnancySkin changes and dermatoses of pregnancy
Skin changes and dermatoses of pregnancy
 

Similar a MCQs Techniques

Cough management issues in pediatric uri - Dr Gaurav Gupta
Cough management issues in pediatric uri - Dr Gaurav GuptaCough management issues in pediatric uri - Dr Gaurav Gupta
Cough management issues in pediatric uri - Dr Gaurav GuptaGaurav Gupta
 
07 what is casuality assesment
07 what is casuality assesment07 what is casuality assesment
07 what is casuality assesmentPrabir Chatterjee
 
Upper respiratory tract infection
Upper respiratory tract infectionUpper respiratory tract infection
Upper respiratory tract infectionNeha Bharti
 
ARI (Community Medicine), PSM, Community Medicine
ARI (Community Medicine), PSM, Community MedicineARI (Community Medicine), PSM, Community Medicine
ARI (Community Medicine), PSM, Community MedicineNepalgunj Medical College
 
Fever without a focus 
(Pediatric Mystery)
Fever without a focus 
(Pediatric Mystery) Fever without a focus 
(Pediatric Mystery)
Fever without a focus 
(Pediatric Mystery) deiaaldeen khudhair
 
Strep Throat
Strep ThroatStrep Throat
Strep Throatfitango
 
GR AFHS COPD.7.8.2020 -FINAL wo CE for ho.pptx
GR AFHS COPD.7.8.2020 -FINAL wo CE for ho.pptxGR AFHS COPD.7.8.2020 -FINAL wo CE for ho.pptx
GR AFHS COPD.7.8.2020 -FINAL wo CE for ho.pptxAFHSResources
 
K. shirin s0204854 educational cd
K. shirin s0204854 educational cdK. shirin s0204854 educational cd
K. shirin s0204854 educational cdshirin_s0204854
 
0042_0044_stephenson.pdf
0042_0044_stephenson.pdf0042_0044_stephenson.pdf
0042_0044_stephenson.pdfsheriftaha22
 
Comics in Pediatric Patient Education - Comics & Medicine 2014
Comics in Pediatric Patient Education - Comics & Medicine 2014Comics in Pediatric Patient Education - Comics & Medicine 2014
Comics in Pediatric Patient Education - Comics & Medicine 2014BoosterShot
 
Non ti resta che cercare, se
Non ti resta che cercare, seNon ti resta che cercare, se
Non ti resta che cercare, seromondia
 
1GastroenteritisNameUnited State University
1GastroenteritisNameUnited State University1GastroenteritisNameUnited State University
1GastroenteritisNameUnited State UniversityAnastaciaShadelb
 
Evidence-based Medicine
Evidence-based MedicineEvidence-based Medicine
Evidence-based Medicineshabeel pn
 
evidence based medical practice
evidence based medical practiceevidence based medical practice
evidence based medical practiceshabeel pn
 

Similar a MCQs Techniques (20)

Cough management issues in pediatric uri - Dr Gaurav Gupta
Cough management issues in pediatric uri - Dr Gaurav GuptaCough management issues in pediatric uri - Dr Gaurav Gupta
Cough management issues in pediatric uri - Dr Gaurav Gupta
 
07 what is casuality assesment
07 what is casuality assesment07 what is casuality assesment
07 what is casuality assesment
 
Upper respiratory tract infection
Upper respiratory tract infectionUpper respiratory tract infection
Upper respiratory tract infection
 
ARI (Community Medicine), PSM, Community Medicine
ARI (Community Medicine), PSM, Community MedicineARI (Community Medicine), PSM, Community Medicine
ARI (Community Medicine), PSM, Community Medicine
 
ARI's and INFLUENZA
ARI's and INFLUENZA ARI's and INFLUENZA
ARI's and INFLUENZA
 
Fever without a focus 
(Pediatric Mystery)
Fever without a focus 
(Pediatric Mystery) Fever without a focus 
(Pediatric Mystery)
Fever without a focus 
(Pediatric Mystery)
 
Mcq ped neuro
Mcq ped neuroMcq ped neuro
Mcq ped neuro
 
April-2020 - ADDRESSING THE USE OF PROPHYLAXIS.docx
April-2020 - ADDRESSING THE USE OF PROPHYLAXIS.docxApril-2020 - ADDRESSING THE USE OF PROPHYLAXIS.docx
April-2020 - ADDRESSING THE USE OF PROPHYLAXIS.docx
 
Strep Throat
Strep ThroatStrep Throat
Strep Throat
 
GR AFHS COPD.7.8.2020 -FINAL wo CE for ho.pptx
GR AFHS COPD.7.8.2020 -FINAL wo CE for ho.pptxGR AFHS COPD.7.8.2020 -FINAL wo CE for ho.pptx
GR AFHS COPD.7.8.2020 -FINAL wo CE for ho.pptx
 
K. shirin s0204854 educational cd
K. shirin s0204854 educational cdK. shirin s0204854 educational cd
K. shirin s0204854 educational cd
 
COMMON COLD IN CHILDREN
COMMON COLD IN CHILDRENCOMMON COLD IN CHILDREN
COMMON COLD IN CHILDREN
 
0042_0044_stephenson.pdf
0042_0044_stephenson.pdf0042_0044_stephenson.pdf
0042_0044_stephenson.pdf
 
Pneumonia
PneumoniaPneumonia
Pneumonia
 
Comics in Pediatric Patient Education - Comics & Medicine 2014
Comics in Pediatric Patient Education - Comics & Medicine 2014Comics in Pediatric Patient Education - Comics & Medicine 2014
Comics in Pediatric Patient Education - Comics & Medicine 2014
 
Okike for web
Okike for webOkike for web
Okike for web
 
Non ti resta che cercare, se
Non ti resta che cercare, seNon ti resta che cercare, se
Non ti resta che cercare, se
 
1GastroenteritisNameUnited State University
1GastroenteritisNameUnited State University1GastroenteritisNameUnited State University
1GastroenteritisNameUnited State University
 
Evidence-based Medicine
Evidence-based MedicineEvidence-based Medicine
Evidence-based Medicine
 
evidence based medical practice
evidence based medical practiceevidence based medical practice
evidence based medical practice
 

Más de FayzaRayes

Steps of Smoking Cessation Badr Bin Himd.pptx
Steps of Smoking Cessation Badr Bin Himd.pptxSteps of Smoking Cessation Badr Bin Himd.pptx
Steps of Smoking Cessation Badr Bin Himd.pptxFayzaRayes
 
DM Saudi Guidelines By DR. Wedad Bardisi.pptx
DM Saudi Guidelines By DR. Wedad Bardisi.pptxDM Saudi Guidelines By DR. Wedad Bardisi.pptx
DM Saudi Guidelines By DR. Wedad Bardisi.pptxFayzaRayes
 
DR. Wedad Bardisi DM Saudi Guideline.pptx
DR. Wedad Bardisi DM Saudi Guideline.pptxDR. Wedad Bardisi DM Saudi Guideline.pptx
DR. Wedad Bardisi DM Saudi Guideline.pptxFayzaRayes
 
DR. Wedad Bardisi DM Saudi Guideline.pptx
DR. Wedad Bardisi DM Saudi Guideline.pptxDR. Wedad Bardisi DM Saudi Guideline.pptx
DR. Wedad Bardisi DM Saudi Guideline.pptxFayzaRayes
 
Introduction FM2 [2014]
Introduction FM2 [2014]Introduction FM2 [2014]
Introduction FM2 [2014]FayzaRayes
 
Introduction FM3 [2014]
Introduction FM3 [2014]Introduction FM3 [2014]
Introduction FM3 [2014]FayzaRayes
 
Illness Behavior IMC الأهلة التفاعل مع المرض 2013
Illness Behavior IMC الأهلة التفاعل مع المرض 2013Illness Behavior IMC الأهلة التفاعل مع المرض 2013
Illness Behavior IMC الأهلة التفاعل مع المرض 2013FayzaRayes
 
Introduction FM2
Introduction FM2Introduction FM2
Introduction FM2FayzaRayes
 
Cholesterol Myth and Truth
Cholesterol Myth and TruthCholesterol Myth and Truth
Cholesterol Myth and TruthFayzaRayes
 
Smoking Counseling
Smoking CounselingSmoking Counseling
Smoking CounselingFayzaRayes
 
Very Special Pictures
Very Special PicturesVery Special Pictures
Very Special PicturesFayzaRayes
 
Migraine and tension headache
Migraine and tension headacheMigraine and tension headache
Migraine and tension headacheFayzaRayes
 
Health Education
Health EducationHealth Education
Health EducationFayzaRayes
 
Health education
Health educationHealth education
Health educationFayzaRayes
 
Applying consultation skills
Applying consultation skillsApplying consultation skills
Applying consultation skillsFayzaRayes
 
Dermatolody quizzes
Dermatolody quizzesDermatolody quizzes
Dermatolody quizzesFayzaRayes
 

Más de FayzaRayes (19)

Steps of Smoking Cessation Badr Bin Himd.pptx
Steps of Smoking Cessation Badr Bin Himd.pptxSteps of Smoking Cessation Badr Bin Himd.pptx
Steps of Smoking Cessation Badr Bin Himd.pptx
 
DM Saudi Guidelines By DR. Wedad Bardisi.pptx
DM Saudi Guidelines By DR. Wedad Bardisi.pptxDM Saudi Guidelines By DR. Wedad Bardisi.pptx
DM Saudi Guidelines By DR. Wedad Bardisi.pptx
 
DR. Wedad Bardisi DM Saudi Guideline.pptx
DR. Wedad Bardisi DM Saudi Guideline.pptxDR. Wedad Bardisi DM Saudi Guideline.pptx
DR. Wedad Bardisi DM Saudi Guideline.pptx
 
DR. Wedad Bardisi DM Saudi Guideline.pptx
DR. Wedad Bardisi DM Saudi Guideline.pptxDR. Wedad Bardisi DM Saudi Guideline.pptx
DR. Wedad Bardisi DM Saudi Guideline.pptx
 
Introduction FM2 [2014]
Introduction FM2 [2014]Introduction FM2 [2014]
Introduction FM2 [2014]
 
Introduction FM3 [2014]
Introduction FM3 [2014]Introduction FM3 [2014]
Introduction FM3 [2014]
 
Illness Behavior IMC الأهلة التفاعل مع المرض 2013
Illness Behavior IMC الأهلة التفاعل مع المرض 2013Illness Behavior IMC الأهلة التفاعل مع المرض 2013
Illness Behavior IMC الأهلة التفاعل مع المرض 2013
 
Joint X-Ray
Joint X-RayJoint X-Ray
Joint X-Ray
 
Urin Analysis
Urin AnalysisUrin Analysis
Urin Analysis
 
Introduction FM2
Introduction FM2Introduction FM2
Introduction FM2
 
Cholesterol Myth and Truth
Cholesterol Myth and TruthCholesterol Myth and Truth
Cholesterol Myth and Truth
 
Smoking Counseling
Smoking CounselingSmoking Counseling
Smoking Counseling
 
Very Special Pictures
Very Special PicturesVery Special Pictures
Very Special Pictures
 
Cartoons
CartoonsCartoons
Cartoons
 
Migraine and tension headache
Migraine and tension headacheMigraine and tension headache
Migraine and tension headache
 
Health Education
Health EducationHealth Education
Health Education
 
Health education
Health educationHealth education
Health education
 
Applying consultation skills
Applying consultation skillsApplying consultation skills
Applying consultation skills
 
Dermatolody quizzes
Dermatolody quizzesDermatolody quizzes
Dermatolody quizzes
 

Último

Hematology and Immunology - Leukocytes Functions
Hematology and Immunology - Leukocytes FunctionsHematology and Immunology - Leukocytes Functions
Hematology and Immunology - Leukocytes FunctionsMedicoseAcademics
 
Call Girls Service Nandiambakkam | 7001305949 At Low Cost Cash Payment Booking
Call Girls Service Nandiambakkam | 7001305949 At Low Cost Cash Payment BookingCall Girls Service Nandiambakkam | 7001305949 At Low Cost Cash Payment Booking
Call Girls Service Nandiambakkam | 7001305949 At Low Cost Cash Payment BookingNehru place Escorts
 
Call Girls Hosur Just Call 7001305949 Top Class Call Girl Service Available
Call Girls Hosur Just Call 7001305949 Top Class Call Girl Service AvailableCall Girls Hosur Just Call 7001305949 Top Class Call Girl Service Available
Call Girls Hosur Just Call 7001305949 Top Class Call Girl Service Availablenarwatsonia7
 
Dwarka Sector 6 Call Girls ( 9873940964 ) Book Hot And Sexy Girls In A Few Cl...
Dwarka Sector 6 Call Girls ( 9873940964 ) Book Hot And Sexy Girls In A Few Cl...Dwarka Sector 6 Call Girls ( 9873940964 ) Book Hot And Sexy Girls In A Few Cl...
Dwarka Sector 6 Call Girls ( 9873940964 ) Book Hot And Sexy Girls In A Few Cl...rajnisinghkjn
 
Call Girls Thane Just Call 9910780858 Get High Class Call Girls Service
Call Girls Thane Just Call 9910780858 Get High Class Call Girls ServiceCall Girls Thane Just Call 9910780858 Get High Class Call Girls Service
Call Girls Thane Just Call 9910780858 Get High Class Call Girls Servicesonalikaur4
 
Call Girls Hebbal Just Call 7001305949 Top Class Call Girl Service Available
Call Girls Hebbal Just Call 7001305949 Top Class Call Girl Service AvailableCall Girls Hebbal Just Call 7001305949 Top Class Call Girl Service Available
Call Girls Hebbal Just Call 7001305949 Top Class Call Girl Service Availablenarwatsonia7
 
97111 47426 Call Girls In Delhi MUNIRKAA
97111 47426 Call Girls In Delhi MUNIRKAA97111 47426 Call Girls In Delhi MUNIRKAA
97111 47426 Call Girls In Delhi MUNIRKAAjennyeacort
 
Call Girl Surat Madhuri 7001305949 Independent Escort Service Surat
Call Girl Surat Madhuri 7001305949 Independent Escort Service SuratCall Girl Surat Madhuri 7001305949 Independent Escort Service Surat
Call Girl Surat Madhuri 7001305949 Independent Escort Service Suratnarwatsonia7
 
Low Rate Call Girls Mumbai Suman 9910780858 Independent Escort Service Mumbai
Low Rate Call Girls Mumbai Suman 9910780858 Independent Escort Service MumbaiLow Rate Call Girls Mumbai Suman 9910780858 Independent Escort Service Mumbai
Low Rate Call Girls Mumbai Suman 9910780858 Independent Escort Service Mumbaisonalikaur4
 
Call Girls In Andheri East Call 9920874524 Book Hot And Sexy Girls
Call Girls In Andheri East Call 9920874524 Book Hot And Sexy GirlsCall Girls In Andheri East Call 9920874524 Book Hot And Sexy Girls
Call Girls In Andheri East Call 9920874524 Book Hot And Sexy Girlsnehamumbai
 
VIP Call Girls Lucknow Nandini 7001305949 Independent Escort Service Lucknow
VIP Call Girls Lucknow Nandini 7001305949 Independent Escort Service LucknowVIP Call Girls Lucknow Nandini 7001305949 Independent Escort Service Lucknow
VIP Call Girls Lucknow Nandini 7001305949 Independent Escort Service Lucknownarwatsonia7
 
Call Girl Service Bidadi - For 7001305949 Cheap & Best with original Photos
Call Girl Service Bidadi - For 7001305949 Cheap & Best with original PhotosCall Girl Service Bidadi - For 7001305949 Cheap & Best with original Photos
Call Girl Service Bidadi - For 7001305949 Cheap & Best with original Photosnarwatsonia7
 
Russian Call Girl Brookfield - 7001305949 Escorts Service 50% Off with Cash O...
Russian Call Girl Brookfield - 7001305949 Escorts Service 50% Off with Cash O...Russian Call Girl Brookfield - 7001305949 Escorts Service 50% Off with Cash O...
Russian Call Girl Brookfield - 7001305949 Escorts Service 50% Off with Cash O...narwatsonia7
 
Glomerular Filtration rate and its determinants.pptx
Glomerular Filtration rate and its determinants.pptxGlomerular Filtration rate and its determinants.pptx
Glomerular Filtration rate and its determinants.pptxDr.Nusrat Tariq
 
Housewife Call Girls Bangalore - Call 7001305949 Rs-3500 with A/C Room Cash o...
Housewife Call Girls Bangalore - Call 7001305949 Rs-3500 with A/C Room Cash o...Housewife Call Girls Bangalore - Call 7001305949 Rs-3500 with A/C Room Cash o...
Housewife Call Girls Bangalore - Call 7001305949 Rs-3500 with A/C Room Cash o...narwatsonia7
 
call girls in munirka DELHI 🔝 >༒9540349809 🔝 genuine Escort Service 🔝✔️✔️
call girls in munirka  DELHI 🔝 >༒9540349809 🔝 genuine Escort Service 🔝✔️✔️call girls in munirka  DELHI 🔝 >༒9540349809 🔝 genuine Escort Service 🔝✔️✔️
call girls in munirka DELHI 🔝 >༒9540349809 🔝 genuine Escort Service 🔝✔️✔️saminamagar
 
Call Girl Bangalore Nandini 7001305949 Independent Escort Service Bangalore
Call Girl Bangalore Nandini 7001305949 Independent Escort Service BangaloreCall Girl Bangalore Nandini 7001305949 Independent Escort Service Bangalore
Call Girl Bangalore Nandini 7001305949 Independent Escort Service Bangalorenarwatsonia7
 
Call Girls Service Noida Maya 9711199012 Independent Escort Service Noida
Call Girls Service Noida Maya 9711199012 Independent Escort Service NoidaCall Girls Service Noida Maya 9711199012 Independent Escort Service Noida
Call Girls Service Noida Maya 9711199012 Independent Escort Service NoidaPooja Gupta
 
Russian Call Girls Gunjur Mugalur Road : 7001305949 High Profile Model Escort...
Russian Call Girls Gunjur Mugalur Road : 7001305949 High Profile Model Escort...Russian Call Girls Gunjur Mugalur Road : 7001305949 High Profile Model Escort...
Russian Call Girls Gunjur Mugalur Road : 7001305949 High Profile Model Escort...narwatsonia7
 
Call Girls Jp Nagar Just Call 7001305949 Top Class Call Girl Service Available
Call Girls Jp Nagar Just Call 7001305949 Top Class Call Girl Service AvailableCall Girls Jp Nagar Just Call 7001305949 Top Class Call Girl Service Available
Call Girls Jp Nagar Just Call 7001305949 Top Class Call Girl Service Availablenarwatsonia7
 

Último (20)

Hematology and Immunology - Leukocytes Functions
Hematology and Immunology - Leukocytes FunctionsHematology and Immunology - Leukocytes Functions
Hematology and Immunology - Leukocytes Functions
 
Call Girls Service Nandiambakkam | 7001305949 At Low Cost Cash Payment Booking
Call Girls Service Nandiambakkam | 7001305949 At Low Cost Cash Payment BookingCall Girls Service Nandiambakkam | 7001305949 At Low Cost Cash Payment Booking
Call Girls Service Nandiambakkam | 7001305949 At Low Cost Cash Payment Booking
 
Call Girls Hosur Just Call 7001305949 Top Class Call Girl Service Available
Call Girls Hosur Just Call 7001305949 Top Class Call Girl Service AvailableCall Girls Hosur Just Call 7001305949 Top Class Call Girl Service Available
Call Girls Hosur Just Call 7001305949 Top Class Call Girl Service Available
 
Dwarka Sector 6 Call Girls ( 9873940964 ) Book Hot And Sexy Girls In A Few Cl...
Dwarka Sector 6 Call Girls ( 9873940964 ) Book Hot And Sexy Girls In A Few Cl...Dwarka Sector 6 Call Girls ( 9873940964 ) Book Hot And Sexy Girls In A Few Cl...
Dwarka Sector 6 Call Girls ( 9873940964 ) Book Hot And Sexy Girls In A Few Cl...
 
Call Girls Thane Just Call 9910780858 Get High Class Call Girls Service
Call Girls Thane Just Call 9910780858 Get High Class Call Girls ServiceCall Girls Thane Just Call 9910780858 Get High Class Call Girls Service
Call Girls Thane Just Call 9910780858 Get High Class Call Girls Service
 
Call Girls Hebbal Just Call 7001305949 Top Class Call Girl Service Available
Call Girls Hebbal Just Call 7001305949 Top Class Call Girl Service AvailableCall Girls Hebbal Just Call 7001305949 Top Class Call Girl Service Available
Call Girls Hebbal Just Call 7001305949 Top Class Call Girl Service Available
 
97111 47426 Call Girls In Delhi MUNIRKAA
97111 47426 Call Girls In Delhi MUNIRKAA97111 47426 Call Girls In Delhi MUNIRKAA
97111 47426 Call Girls In Delhi MUNIRKAA
 
Call Girl Surat Madhuri 7001305949 Independent Escort Service Surat
Call Girl Surat Madhuri 7001305949 Independent Escort Service SuratCall Girl Surat Madhuri 7001305949 Independent Escort Service Surat
Call Girl Surat Madhuri 7001305949 Independent Escort Service Surat
 
Low Rate Call Girls Mumbai Suman 9910780858 Independent Escort Service Mumbai
Low Rate Call Girls Mumbai Suman 9910780858 Independent Escort Service MumbaiLow Rate Call Girls Mumbai Suman 9910780858 Independent Escort Service Mumbai
Low Rate Call Girls Mumbai Suman 9910780858 Independent Escort Service Mumbai
 
Call Girls In Andheri East Call 9920874524 Book Hot And Sexy Girls
Call Girls In Andheri East Call 9920874524 Book Hot And Sexy GirlsCall Girls In Andheri East Call 9920874524 Book Hot And Sexy Girls
Call Girls In Andheri East Call 9920874524 Book Hot And Sexy Girls
 
VIP Call Girls Lucknow Nandini 7001305949 Independent Escort Service Lucknow
VIP Call Girls Lucknow Nandini 7001305949 Independent Escort Service LucknowVIP Call Girls Lucknow Nandini 7001305949 Independent Escort Service Lucknow
VIP Call Girls Lucknow Nandini 7001305949 Independent Escort Service Lucknow
 
Call Girl Service Bidadi - For 7001305949 Cheap & Best with original Photos
Call Girl Service Bidadi - For 7001305949 Cheap & Best with original PhotosCall Girl Service Bidadi - For 7001305949 Cheap & Best with original Photos
Call Girl Service Bidadi - For 7001305949 Cheap & Best with original Photos
 
Russian Call Girl Brookfield - 7001305949 Escorts Service 50% Off with Cash O...
Russian Call Girl Brookfield - 7001305949 Escorts Service 50% Off with Cash O...Russian Call Girl Brookfield - 7001305949 Escorts Service 50% Off with Cash O...
Russian Call Girl Brookfield - 7001305949 Escorts Service 50% Off with Cash O...
 
Glomerular Filtration rate and its determinants.pptx
Glomerular Filtration rate and its determinants.pptxGlomerular Filtration rate and its determinants.pptx
Glomerular Filtration rate and its determinants.pptx
 
Housewife Call Girls Bangalore - Call 7001305949 Rs-3500 with A/C Room Cash o...
Housewife Call Girls Bangalore - Call 7001305949 Rs-3500 with A/C Room Cash o...Housewife Call Girls Bangalore - Call 7001305949 Rs-3500 with A/C Room Cash o...
Housewife Call Girls Bangalore - Call 7001305949 Rs-3500 with A/C Room Cash o...
 
call girls in munirka DELHI 🔝 >༒9540349809 🔝 genuine Escort Service 🔝✔️✔️
call girls in munirka  DELHI 🔝 >༒9540349809 🔝 genuine Escort Service 🔝✔️✔️call girls in munirka  DELHI 🔝 >༒9540349809 🔝 genuine Escort Service 🔝✔️✔️
call girls in munirka DELHI 🔝 >༒9540349809 🔝 genuine Escort Service 🔝✔️✔️
 
Call Girl Bangalore Nandini 7001305949 Independent Escort Service Bangalore
Call Girl Bangalore Nandini 7001305949 Independent Escort Service BangaloreCall Girl Bangalore Nandini 7001305949 Independent Escort Service Bangalore
Call Girl Bangalore Nandini 7001305949 Independent Escort Service Bangalore
 
Call Girls Service Noida Maya 9711199012 Independent Escort Service Noida
Call Girls Service Noida Maya 9711199012 Independent Escort Service NoidaCall Girls Service Noida Maya 9711199012 Independent Escort Service Noida
Call Girls Service Noida Maya 9711199012 Independent Escort Service Noida
 
Russian Call Girls Gunjur Mugalur Road : 7001305949 High Profile Model Escort...
Russian Call Girls Gunjur Mugalur Road : 7001305949 High Profile Model Escort...Russian Call Girls Gunjur Mugalur Road : 7001305949 High Profile Model Escort...
Russian Call Girls Gunjur Mugalur Road : 7001305949 High Profile Model Escort...
 
Call Girls Jp Nagar Just Call 7001305949 Top Class Call Girl Service Available
Call Girls Jp Nagar Just Call 7001305949 Top Class Call Girl Service AvailableCall Girls Jp Nagar Just Call 7001305949 Top Class Call Girl Service Available
Call Girls Jp Nagar Just Call 7001305949 Top Class Call Girl Service Available
 

MCQs Techniques

  • 1. Fayza Rayes MBBCh. MSc. MRCGP Consultant Family Physician Joint Program of Family & Community Medicine, Jeddah www.fayzarayes.com
  • 2. Objectives: 1. Improving your out come from studding 2. Improving your technique in answering MCQs
  • 3. Introduction: 1. Do you follow any techniques while studying from MCQ book 2. Do you follow any techniques while answering MCQs questions
  • 4. Contents:  10 tips in studying for MCQs exam  10 tips in answering MCQs  Answering 45 MCQs
  • 5. 10 Tips in studying MCQs: 1. Write your studying plan 2. Study in a group 3. Answering one MCQ is an opportunity to review lots of knowledge . Make sure to get the maximum from every single MCQ . Read the explanation and review the related topic 4. Write a short abstract about each important topic 5. Write table to compare between similar diseases 6. Be specific in your information and extra specific
  • 6. How can you benefit from this information ?
  • 7. 10 Tips in studying MCQs: 7. Concentrate on important topics in family medicine e.g.:  Children growth and development  Vaccination  Presentation of common diseases special patients. e.g. appendicitis in very young and in very elderly patients  Prescribing skills of antibiotics and psychotropic medications
  • 8. 10 Tips in studying MCQs: 8. To improve your mental fitness, periodically take mock exam, start by 10 questions then increase it gradually till you take full mock exam (100-150 MCQs in 2-3 hours continuously) 9. Revise your mistake periodically (some mistakes are false believes… like delusion very difficult to correct ) 10. Just before the exam, you need at least 2-3 weeks for revision (revise your mistakes , read your personal abstracts )
  • 9. Correction of false believes: Do you have any strategy?? 1. Writing summaries 2. Teaching others the new information 3. Reminding cards 4. …….
  • 10. How to write your valuable summery 1. Typical presentation of the disease 2. Pathognomonic features 3. Areas of similarity and areas of difference from other differential diagnosis (to make the question difficult) 4. Initial investigation (cost effectiveness and sensitivity) 5. Diagnostic test (more specific) 6. Initial management (non-pharmacological + or -) 7. Most effective management (It could be costly , it could have more side effect) 8. New update ??!!
  • 11. 10 Tips in answering MCQs: 1. Pay attention to the key words 2. When you chose an answer make sure the stem of the MCQ and the answer you have chosen are grammatically correct If you are not sure of the answer make intelligent guising: 3. Find your correct answer by process of exclusion:  First delete the apparently incorrect answer  Any answer partially incorrect delete it from your choices  If two choices are similar both are incorrect  If two choices are opposite to each other , one of them is the correct answer  If one choice is different from the rest, consider it the right answer
  • 12. 10 Tips in answering MCQs: 4. Some MCQs ask about number . Learn how to deal with numbers  Arrange the answers in order – lowest->> highest  Eliminate the lowest and highest number  Go with the general theme of family medicine practice  Chose the meaningful number 5. In general, respect your logical thinking & your clinical experience
  • 13. 10 Tips in answering MCQs: Examples logical thinking:  Smoking is a risk factors of nearly any health problem  Old medications have more side effects than new medications  In general drug with less side effect are with weaker effect, and the vise versa  To change to any new intervention (drug, procedure, investigation or vaccine …) the effect of the new intervention must be the same or better effects and with less side effects
  • 14. 10 Tips in answering MCQs: 6. In controversial issues, respect what you can remember from your MCQs reference books 7. Don’t postponed writing in the answer sheet till near the end of the exam time 8. Don’t hand over your paper early . Use your time fully 9. Near the end of exam time, you need 5-10 minutes for revision to avoid stupid mistake 10. Do not change any answer unless you are 100% sure that your first choice was wrong
  • 15. A 6-month-old infant is brought into your office by his mother. She presents with 2-day history of wheezing, a mild fever (38.5°C), and rhinorrhea. The child has no known allergies but there is a history of allergies in the family. On examination, the child’s respiratory rate is 60/minute. There is rhonchi and moist rales heard throughout the chest. The chest x-ray shows evidence of hyperaeration. The most likely diagnosis in the child is: A. Mycoplasma pneumoniae infection B. Allergic bronchitis C. Viral tracheitis D. Bacterial tracheitis E. Bronchiolitis .1
  • 16. Key words: A 6-month-old infant is brought into your office by his mother. She presents with 2-day history of wheezing, a mild fever (38.5°C), and rhinorrhea. The child has no known allergies but there is a history of allergies in the family. On examination, the child’s respiratory rate is 60/minute. There is rhonchi and moist rales heard throughout the chest. The chest x- ray shows evidence of hyperaeration.
  • 17. A 6-month-old infant is brought into your office by his mother. She presents with 2-day history of wheezing, a mild fever (38.5°C), and rhinorrhea. The child has no known allergies but there is a history of allergies in the family. On examination, the child’s respiratory rate is 60/minute. There is rhonchi and moist rales heard throughout the chest. The chest x-ray shows evidence of hyperaeration. The most likely diagnosis in the child is: A. Mycoplasma pneumoniae infection B. Allergic bronchitis C. Viral tracheitis D. Bacterial tracheitis E. Bronchiolitis
  • 18.  Bronchiolitis (RSV) In most “typical” kids, this virus is what most parents consider “just a cold,” but it has the potential to become serious, so here’s what you should know  Symptoms: Runny nose, fever, cough, wheezing with breathing, irritability, & loss of appetite.  Treatment  Mild symptoms are treated with rest, fluids, and a cool air humidifier.  Babies who are struggling to breath may hospitalized and given supplemental humidified oxygen. Their breathing will be monitored and if necessary fluids will be given intravenously to prevent dehydration.  Occasionally infants need mechanical ventilation to fill and empty the lungs until the airways open. 1. Example of Quick Revision Bronchiolitis in infants
  • 19. 2. Example of Quicker Revision
  • 20. 1. Infants 2. Wheezing 3. Rhonchi and moist rales 4. Respiratory syncytial virus 5. Cold, humidified, oxygen 3. Example of Abstracts Bronchiolitis in infants
  • 21. A 5-year-old child is brought to the ER with his mother. The mother tells you that for the past 24 hours the child bas been “talking strangely” and drooling. He has had no appetite and has not been drinking. Based on this history, what is the diagnosis of major concern? A. Viral pneumonia B. Acute epiglottis C. Bronchiolitis D. Croup E. Bacterial pneumonia .2
  • 22. A 5-year-old child is brought to the ER with his mother. The mother tells you that for the past 24 hours the child bas been “talking strangely” and drooling. He has had no appetite and has not been drinking. Based on this history, what is the diagnosis of major concern? A. Viral pneumonia B. Acute epiglottis C. Bronchiolitis D. Croup E. Bacterial pneumonia
  • 23. 1. Example of Quick Revision
  • 24. 2. Example of Quicker Revision
  • 26. 1. ER 2. Delusion “talking strangely” 3. Drooling 4. A lateral x-ray of the neck 5. IV/IM ceftriaxone Epiglottitis: 3. Example of Abstracts
  • 27. An 18-month-old infant is brought to the ER by his mother. He developed an upper respirator tract infection 2 days ago and suddenly this evening developed a harsh, barky cough and difficulty breathing. On examination the child is coughing. His respiratory rate is 40/minute and he is in some respiratory distress. The breath sounds that are heard appear to be transmitted from the upper airway. There are nasal flaring and suprasternal, infrasternal, and intercostals retractions. The child’s temperature is 38.5°C.
  • 28. What is the most likely diagnosis in this child? A. Viral pneumonia B. Acute epiglottis C. Bronchiolitis D. Croup E. Bacterial pneumonia .3
  • 29. What is the most likely diagnosis in this child? A. Viral pneumonia - LRTI B. Acute epiglottis - LRTI C. Bronchiolitis - LRTI D. Croup - URTI E. Bacterial pneumonia - LRTI
  • 30. Acute Larynotracheitis (Viral Croup) Differential Diagnosis: The differential diagnosis of viral croup includes epiglottitis, foreign body, and angioneurotic edema. Symptoms: Barking cough – especially at night; high pitched whistling noise with inhalation (stridor); runny nose; fever. Management: Most children with croup do not require hospitalization. Treatment at home consists of air humidification, avoidance of agitation, and reduction of fever If signs of severe obstruction develop, treatment with an aerosol of racemic epinephrine (2.25%), nebulized with 100% oxygen, frequently provides relief. Frequent aerosol treatments may be needed for the first few hours. A single parenteral dose of dexamethasone, 0.6 mg/kg, is effective in decreasing the length and severity of respiratory symptoms that are associated with viral croup. Inhaled corticosteroid therapy has also been used with some success, but parenteral dexamethasone is somewhat easier. Intubation or tracheostomy rarely is necessary. 1. Example of Quick Revision
  • 31. A systematic review of clinical trials confirmed the observations that nebulised adrenaline produces immediate and often dramatic improvements in reducing the signs of airway obstruction in severe croup. Acute Larynotracheitis (Viral Croup) 2. Update & EBM
  • 32. 1. Early childhood 2. ER 3. Harsh, barky cough 4. Respiratory distress 5. Parainfluenza virus 6. Aerosolized steroids 7. Nebulised adrenaline Acute Larynotracheitis (Viral Croup) 3. Example of Abstracts
  • 33. Tips in studying MCQs:  Answering one MCQ is an opportunity to review lots of knowledge . Make sure to get the maximum from every single MCQ . Read the explanation and review the related topic  Write a short abstract about each important topic A. Write pathognomonic features B. Write Update information C. Write controversial issues AND stick to your NOTES  To master clear differentiation make comparisons between similar issues
  • 34. A 25-year-old college student presents with a 3-week history of fatigue, malaise, fever, chills, and sore throat. On physical examination, the patient has a temperature of 39°C. There is pharyngeal hyperemia and edema and marked exudates are seen in both tonsillar areas. There is significant cervical lymphadenopathy present. You suspect infectious mononucleosis. Of the following clinical features of acute infectious mononucleosis, the least common is: A. Splenomegaly B. Hepatomegaly C. Fever D. Exudative tonsillitis E. Generalized lymphadenopathy .4
  • 35. Key words A 25-year-old college student presents with a 3-week history of fatigue, malaise, fever, chills, and sore throat. On physical examination, the patient has a temperature of 39°C. There is pharyngeal hyperemia and edema and marked exudates are seen in both tonsillar areas. There is significant cervical lymphadenopathy present. You suspect infectious mononucleosis. Of the following clinical features of acute infectious mononucleosis, the least common is: A. Splenomegaly – (30-45%) B. Hepatomegaly – (Impaired LFTs --30% of the patients) C. Fever - 85% D. Exudative tonsillitis – 60% E. Generalized lymphadenopathy – 85%
  • 36. Breast Cancer A 41-year-old woman comes to your office after finding a breast lump during a routine self-examination. She has been examining her breasts regularly for the past 5 years; this is the first lump she has found. On examination, there is a lump located in the right breast. The lump’s anatomic location is in the upper outer quadrant. It is approximately 3 cm in diameter and is not fixed to skin or muscle. It has a hard consistency. There are three axillary nodes present on the right side; each node is approximately 1 cm in diameter. No lymph nodes are present on the left. At this time, what would you do? A. tell the patient that she has fibrocystic breast disease; ask her to return in 1 month, preferably 10 days after the next period, for a recheck B tell the patient to come back for a breast examination in 6 months C. order mammogram D. order an ultrasound examination of the area E. arrange for urgent fine-needle biopsy .5
  • 37.
  • 38. Breast Cancer A 41-year-old woman comes to your office after finding a breast lump during a routine self-examination. She has been examining her breasts regularly for the past 5 years; this is the first lump she has found. On examination, there is a lump located in the right breast. The lump’s anatomic location is in the upper outer quadrant. It is approximately 3 cm in diameter and is not fixed to skin or muscle. It has a hard consistency. There are three axillary nodes present on the right side; each node is approximately 1 cm in diameter. No lymph nodes are present on the left. At this time, what would you do? A. tell the patient that she has fibrocystic breast disease; ask her to return in 1 month, preferably 10 days after the next period, for a recheck B tell the patient to come back for a breast examination in 6 months C. order mammogram D. order an ultrasound examination of the area E. arrange for urgent fine-needle biopsy
  • 39. What do we mean by being specific?! General information X Detailed information
  • 40. What do we mean by being specific?! General information X Detailed information This is general information
  • 41. What do we mean by being specific?! General information X Detailed information This is detailed information
  • 42. Tips in studying MCQs:  Be specific in your information  And be extra specific and meticulous
  • 43. Immune-deficient Child Which one of the following vaccine must not be given to a household contact of an immune-deficient child? A. Mumps, measles and rubella. B. BCG C. Influenza vaccine D. Oral polio vaccine E. Hepatitis B vaccine .6
  • 44. Immune-deficient Child Which one of the following vaccine must not be given to a household contact of an immune-deficient child? A. Mumps, measles and rubella. B. BCG C. Influenza vaccine D. Oral polio vaccine E. Hepatitis B vaccine
  • 45. Breast Feeding Which of the following statements about breast-feeding is true? A) The infant should feed on each side for 8 to 15 minutes every 2 to 3 hours after birth. B) Colostrum is excreted 7 to 10 days after delivery and contains important antibodies, high calories, and other nutrients. C) The mother should weigh infants before and after feeding to quantify the amount consumed. D) Breast-feeding usually provides adequate nutrition for 2 to 4 months—supplementation should begin at that point. E) Breast-feeding should be based on timed intervals rather than on demand. .7
  • 46. Breast Feeding Which of the following statements about breast-feeding is true? A) The infant should feed on each side for 8 to 15 minutes every 2 to 3 hours after birth. B) Colostrum is excreted 7 to 10 days after delivery and contains important antibodies, high calories, and other nutrients. C) The mother should weigh infants before and after feeding to quantify the amount consumed. D) Breast-feeding usually provides adequate nutrition for 2 to 4 months—supplementation should begin at that point. E) Breast-feeding should be based on timed intervals rather than on demand. The answer is A. Breast-feeding is encouraged for all mothers . Currently, as many as 50% of mothers.
  • 47. Growth & Development An 8-month-old infant is brought to the emergency department by his mother for an assessment of an upper respiratory tract infection. He has been coughing for the past 3 days and has had a runny nose. On examination, his temperature is 37.5°C. His weight is below the 3rd percentile for his age, his length is at the 25th percentile, and his head circumference is at the 50th percentile. He appears malnourished and has thin extremities, a narrow face, prominent ribs, and wasted buttocks. He has a prominent diaper rash, unwashed skin, a skin rash that resembles the skin infection impetigo contagious on his face, uncut fingernails, and dirty clothing.  What is the most likely cause of this child’s condition?  a. maternal deprivation  b. cystic fibrosis  c. constitutionally small for age  d. infantile autism  e. congenital bilateral sensorineural hearing loss .8
  • 48. Growth & Development An 8-month-old infant is brought to the emergency department by his mother for an assessment of an upper respiratory tract infection. He has been coughing for the past 3 days and has had a runny nose. On examination, his temperature is 37.5°C. His weight is below the 3rd percentile for his age, his length is at the 25th percentile, and his head circumference is at the 50th percentile. He appears malnourished and has thin extremities, a narrow face, prominent ribs, and wasted buttocks. He has a prominent diaper rash, unwashed skin, a skin rash that resembles the skin infection impetigo contagiosum on his face, uncut fingernails, and dirty clothing.  What is the most likely cause of this child’s condition?  a. maternal deprivation  b. cystic fibrosis  c. constitutionally small for age  d. infantile autism  e. congenital bilateral sensorineural hearing loss
  • 49. Growth & Development Bone age can sometimes be used to differentiate certain causes of short stature in children. With respect to bone age, which of the following statements is true? A. bone age is normal in both familial short stature and constitutional delay of growth B. bone age is normal in familial short stature and delayed in constitutional delay of growth C. bone age is normal in constitutional delay of growth and delayed in growth hormone deficiency D. bone age is delayed in both familial short stature and short stature caused by hypothyroidism E. bone age is variable and cannot be used to differentiate familial short stature and constitutional delay .9
  • 50. Growth & Development Bone age can sometimes be used to differentiate certain causes of short stature in children. With respect to bone age, which of the following statements is true? A. bone age is normal in both familial short stature and constitutional delay of growth B. bone age is normal in familial short stature and delayed in constitutional delay of growth C. bone age is normal in constitutional delay of growth and delayed in growth hormone deficiency D. bone age is delayed in both familial short stature and short stature caused by hypothyroidism E. bone age is variable and cannot be used to differentiate familial short stature and constitutional delay
  • 51. Teeth Eruption A 9-month-old boy brought by his mother to see you in the clinic. Her main concern that her son did not have teeth eruption till now. He is developmentally normal. His height and weight are normal. What will be your advice to her? A. Tell her don't worry except after 1 month. B. It is not unusual for the first deciduous teeth to erupt before 12 months. C. Investigate for hypothyroidism. D. Do roentgenogram for mandible and maxillary X ray. E. Tell her that because her son have a nails, so he will also have teeth eruption. .10
  • 52. Teeth Eruption A 9-month-old boy brought by his mother to see you in the clinic. Her main concern that her son did not have teeth eruption till now. He is developmentally normal. His height and weight are normal. What will be your advice to her? A. Tell her don't worry except after 1 month. B. It is not unusual for the first deciduous teeth to erupt before 12 months. C. Investigate for hypothyroidism. D. Do roentgenogram for mandible and maxillary X ray. E. Tell her that because her son have a nails, so he will also have teeth eruption.
  • 53. Visual Screening Which statement regarding visual screening in children is correct? A. Visual screening is not indicated until age 5. B. Visual acuity can be assessed by the Random Dot E test. C. Stereopsis can be measured by the Tumbling E test. D. Strabismus can be assessed with the cover test during the first year of life. E. Visual screening can be reliably assessed at age 2. .11
  • 54. Visual Screening Which statement regarding visual screening in children is correct? A. Visual screening is not indicated until age 5. B. Visual acuity can be assessed by the Random Dot E test. C. Stereopsis can be measured by the Tumbling E test. D. Strabismus can be assessed with the cover test during the first year of life. E. Visual screening can be reliably assessed at age 2.
  • 55.
  • 56. Influenza Vaccination Which one of the following patients should not receive an influenza vaccination? A. A 36-year-old pregnant woman at 12 weeks' gestation. B. A 65-year-old man with diabetes mellitus. C. A 28-year-old man with HIV infection. D. A 54-year-old woman who is allergic to eggs. E. A six-year-old girl who lives with a grandmother who has terminal cancer. .12
  • 57. Influenza Vaccination Which one of the following patients should not receive an influenza vaccination? A. A 36-year-old pregnant woman at 12 weeks' gestation. B. A 65-year-old man with diabetes mellitus. C. A 28-year-old man with HIV infection. D. A 54-year-old woman who is allergic to eggs. E. A six-year-old girl who lives with a grandmother who has terminal cancer. Chose the different
  • 58. Knowledge of Vaccination A 68-year-old patient is seen for a general examination. Current recommendations for immunizations include A. tetanus booster every 5 years B. influenza vaccination yearly C. pneumococcal vaccination yearly D. hepatitis booster every 5 years E. meningococcal vaccination .13
  • 59. Knowledge of Vaccination A 68-year-old patient is seen for a general examination. Current recommendations for immunizations include A. tetanus booster every 5 years B. influenza vaccination yearly C. pneumococcal vaccination yearly D. hepatitis booster every 5 years E. meningococcal vaccination
  • 60. Tips in studying MCQs:  Concentrate on important topics in family medicine e.g.:  Children growth and development  Breast feeding  Vaccination  Presentation of common diseases special patients. e.g. appendicitis in very young and in very elderly patients You will be asked in this topics in any FM exam
  • 61. Forgetfulness A 57-year-old male has become forgetful, preoccupied, withdrawn and suspicious. His physical examination was normal. The patient has been with his company for 22 years and was considered an excellent employee. Which of the following is most likely diagnosis? A. Multi-infarct dementia. B. Depression. C. Vascular dementia D. Alcoholism. E. Alzheimer .14
  • 62. Forgetfulness A 57-year-old male has become forgetful, preoccupied, withdrawn and suspicious. His physical examination was normal. The patient has been with his company for 22 years and was considered an excellent employee. Which of the following is most likely diagnosis? A. Multi-infarct dementia. B. Depression. C. Vascular dementia D. Alcoholism. E. Alzheimer
  • 63.
  • 64. Tips in answering MCQs: If you are not sure of the answer make inelegant guising: Find your correct answer by process of exclusion:  First delete the apparently incorrect newer  If two choices are similar both are incorrect
  • 65. Post MI Medications Which of the following improve survival after myocardial infarction in hypertensive patient? A. Nitrate. B. Morphine. C. ACE inhibitors. D. Calcium channel blockers. E. Aspirin. .15
  • 66. Post MI Medications Which of the following improve survival after myocardial infarction in hypertensive patient? A. Nitrate. B. Morphine. C. ACE inhibitors. D. Calcium channel blockers. E. Aspirin.
  • 67. H. Pylori Testing .16 Which of the following types of H. pylori testing is not useful to confirm eradication? A. stool antigen test B. urea breath test C. enzyme-linked immunosorbent assay (ELISA) D. Serology culture E. Steiner stain of gastric biopsy specimen
  • 68. H. Pylori Testing Which of the following types of H. pylori testing is not useful to confirm eradication? A. stool antigen test B. urea breath test C. enzyme-linked immunosorbent assay (ELISA) D. Serology culture E. Steiner stain of gastric biopsy specimen
  • 69. Gout Which one of these drug is not associated with gout? A. Diuretics B. Digoxin C. Niacin (vitamin B3) D. Aspirin E. Cyclosporine (Immunosuppressive) .17
  • 70. Gout Which one of these drug is not associated with gout? A. Diuretics B. Digoxin C. Niacin (vitamin B3) D. Aspirin E. Cyclosporine (Immunosuppressive)
  • 71. Vague Chest Pain  A 35-year-old male consults you about vague chest pain he developed while sitting at his desk earlier in the day. The pain is right-sided and was sharp for a brief time when it began, but it rapidly subsided. There was no hemoptysis and the pain does not seem pleuritic. His physical examination, EKG, and oxygen saturation are unremarkable. A chest film shows a 10% right pneumothorax. Which one of the following should you do next? A. Admit the patient to the hospital for observation. B. Admit the patient to the hospital for chest tube placement. C. Obtain a repeat chest radiograph in 24–48 hours. D. Obtain an expiratory chest radiograph. E. Reassure the patient and do nothing .18
  • 72.
  • 73. Vague Chest Pain  A 35-year-old male consults you about vague chest pain he developed while sitting at his desk earlier in the day. The pain is right-sided and was sharp for a brief time when it began, but it rapidly subsided. There was no hemoptysis and the pain does not seem pleuritic. His physical examination, EKG, and oxygen saturation are unremarkable. A chest film shows a 10% right pneumothorax. Which one of the following should you do next? A. Admit the patient to the hospital for observation. B. Admit the patient to the hospital for chest tube placement. C. Obtain a repeat chest radiograph in 24–48 hours. D. Obtain an expiratory chest radiograph. E. Reassure the patient and do nothing
  • 74. Fatigue and Weakness A 60-year-old man is being evaluated for fatigue, weakness, and exercise intolerance. Laboratory assessment reveals: Hemoglobin: 9.1 mg/dL (L) Serum iron: 46 μg/dL (L) Ferritin: 9 ng/mL (L) Total iron binding capacity (TIBC): 626 μg/dL (H) Mean corpuscular volume (MCV): 76 fL (L) What is the most common cause of this condition? A. Blood loss B. Poor nutrition C. Inadequate absorption of iron D. Chronic disease E. Folic acid deficiency .19
  • 75. Fatigue and Weakness A 60-year-old man is being evaluated for fatigue, weakness, and exercise intolerance. Laboratory assessment reveals: Hemoglobin: 9.1 mg/dL (L) Serum iron: 46 μg/dL (L) Ferritin: 9 ng/mL (L) Total iron binding capacity (TIBC): 626 μg/dL (H) Mean corpuscular volume (MCV): 76 fL (L) What is the most common cause of this condition? A. Blood loss B. b. Poor nutrition C. c. Inadequate absorption of iron D. d. Chronic disease E. e. Folic acid deficiency
  • 76. Liver Disease with Cirrhosis  You are seeing a patient who has end-stage liver disease with cirrhosis due to hepatitis C. What will be her most likely cause of death? A. Liver failure. B. Myocardial infarction. C. Bleeding varices. D. Hepatocellular carcinoma. E. Renal failure. .20
  • 77. Liver Disease with Cirrhosis  You are seeing a patient who has end-stage liver disease with cirrhosis due to hepatitis C. What will be her most likely cause of death? A. Liver failure. B. Myocardial infarction. C. Bleeding varices. D. Hepatocellular carcinoma. E. Renal failure. The main causes of 436 deaths among 532 patients with cirrhosis followed up for up to 16 years constituted liver failure (24%), liver failure with gastrointestinal bleeding (13%), gastrointestinal bleeding (14%) i.e. bleeding ±hepatic failure (27%), primary liver cell carcinoma (4%), other liver-related causes (2%), infections (7%), cardiovascular diseases (22%), extra hepatic malignancies (9%), and other non-liver-related causes (5%). Totally, 57% died of liver-related causes .20
  • 78.
  • 79. Liver Disease with Cirrhosis  You are seeing a patient who has end-stage liver disease with cirrhosis due to hepatitis C. What will be her most likely cause of death? A. Liver failure. B. Myocardial infarction. C. Bleeding varices. D. Hepatocellular carcinoma. E. Renal failure. The main causes of 436 deaths among 532 patients with cirrhosis followed up for up to 16 years constituted liver failure (24%), liver failure with gastrointestinal bleeding (13%), gastrointestinal bleeding (14%) i.e. bleeding ±hepatic failure (27%), primary liver cell carcinoma (4%), other liver-related causes (2%), infections (7%), cardiovascular diseases (22%), extra hepatic malignancies (9%), and other non-liver-related causes (5%). Totally, 57% died of liver-related causes
  • 80. Prostate Cancer The best management of localized, well-differentiated prostate cancer in men older than 65 is: A. Radiation implants. B. External beam radiation therapy. C. Watchful waiting. D. Primary androgen deprivation therapy. E. Robot-assisted prostatectomy. .21
  • 81.
  • 82. Prostate Cancer The best management of localized, well-differentiated prostate cancer in men older than 65 is: A. Radiation implants. B. External beam radiation therapy. C. Watchful waiting. D. Primary androgen deprivation therapy. E. Robot-assisted prostatectomy.
  • 83. Tips in studying MCQs:  Be specific in your information  And be extra specific and meticulous
  • 84. AAA Which one of the following is the greatest risk factor for abdominal aortic aneurysm (AAA)? A. Cigarette smoking. B. Diabetes mellitus. C. Hypertension. D. African-American race. E. Female gender. .22
  • 85. AAA Which one of the following is the greatest risk factor for abdominal aortic aneurysm (AAA)? A. Cigarette smoking. B. Diabetes mellitus. C. Hypertension. D. African-American race. E. Female gender.
  • 86. Abdominal Aortic Aneurysm (AAA)  In the UK the rate of Abdominal Aortic Aneurism (AAA) in Caucasian men older than 65 years is about 4.7%  The peak incidence is among males around 70 years of age, the prevalence among males over 60 years totals 2- 6%.  In the U.S. the incidence of AAA is 2-4% in the adult population.  AAA is 4-6 times more common in male siblings of known patients, with a risk of 20-30%.  Rupture of the AAA occurs in 1-3% of men aged 65 or more, the mortality is 70-95%.
  • 88. Cervical Cancer Which of the following statements regarding cervical cancer screening is true? A. The death rate from cervical cancer continues to increase despite Pap smear screening. B. Immunosuppression has not been identified as a risk factor for cervical cancer. C. Human papillomavirus (HPV) types 1 and 3 are most closely linked to cervical cancer. D. Smoking has been linked to the development of cervical cancer. E. Most cases of cervical cancer occur between the ages of 20 and 30 years. .23
  • 89. Cervical Cancer Which of the following statements regarding cervical cancer screening is true? A. The death rate from cervical cancer continues to increase despite Pap smear screening. B. Immunosuppression has not been identified as a risk factor for cervical cancer. C. Human papillomavirus (HPV) types 1 and 3 are most closely linked to cervical cancer. D. Smoking has been linked to the development of cervical cancer. E. Most cases of cervical cancer occur between the ages of 20 and 30 years.  The answer is D.  Tip: Smoking always the first to blame
  • 90.
  • 91. Tips in answering MCQs: Examples logical thinking: Smoking is the first to blame … Smoking is “ criminal ”
  • 93.  In the British Journal of Psychiatry, the researchers wrote, “The best-fitting causal model was one in which nicotine dependence led to increased risk of depression.” They suggest two possible routes, one involving common risk factors, and the second a direct causal link.  According to the researchers, “this evidence is consistent with the conclusion that there is a cause and effect relationship between smoking and depression in which cigarette smoking increases the risk of symptoms of depression.”  http://psychcentral.com/lib/can-smoking-cause-depression/0007153 Smoking and Depression
  • 94. Smoking Which of the following conditions is NOT associated with smoking? A. Peptic ulcer disease B. Depression C. Children of smokers are at increased risk for otitis media D. Osteoporosis E. Cervical cancer .24
  • 95. Smoking Which of the following conditions is NOT associated with smoking? A. Peptic ulcer disease B. Depression C. Children of smokers are at increased risk for otitis media D. Osteoporosis E. Cervical cancer
  • 96. Tips in answering MCQs:  In general, respect your clinical experience  In controversial issues, respect what you can remember from your MCQs reference books
  • 97. Hepatitis C & Brest Milk Which of the following statements is correct concerning hepatitis C virus (HCV)? A. There is no risk to infants if the mother is affected. B. There is no risk associated with sexual intercourse with an individual with hepatitis C. C. Cesarean section should be performed on mothers who test positive for hepatitis C to prevent transmission to the newborn. D. Hepatitis C can be spread by contaminated water supplies. E. Hepatitis C does not appear to be transmitted in breast milk. .25
  • 98. Hepatitis C & Brest Milk Which of the following statements is correct concerning hepatitis C virus (HCV)? A. There is no risk to infants if the mother is affected. B. There is no risk associated with sexual intercourse with an individual with hepatitis C. C. Cesarean section should be performed on mothers who test positive for hepatitis C to prevent transmission to the newborn. D. Hepatitis C can be spread by contaminated water supplies. E. Hepatitis C does not appear to be transmitted in breast milk.
  • 99. 1. Learn to write very clear message to correct any false believes in your MCQs Knowledge 2. It is more helpful if you add new relevant information to maximize your benefit from MCQs studying
  • 100. Tips in studying MCQs:  Answering one MCQ is an opportunity to review lots of knowledge . Make sure to get the maximum from every single MCQ . Read the explanation and review the related topic  Write a short abstract about each important topic
  • 101. Headache Red Flags Each of the following is a solid indication for neuroimaging in a patient with headache except: A. Onset of headaches over the age of 50 years B. Seizures associated C. Prolonged aura D. Nausea and vomiting E. Headache worsening with movement .26
  • 102. Headache Red Flags Each of the following is a solid indication for neuroimaging in a patient with headache except: A. Onset of headaches over the age of 50 years B. Seizures associated C. Prolonged aura D. Nausea and vomiting E. Headache worsening with movement
  • 103. Head Lice Which of the following is true regarding management of head lice? A. Children should be kept out of school until no visible evidence of nits is noted. B. Household members should only be treated if live lice or eggs are noted within 1 cm of the scalp. C. Head lice programs have had a significant impact on lowering the incidence of head lice. D. Cleaning of bedding has little impact on lice eradication. E. The health of those exposed is more important than the confidentiality of the child affected. .27
  • 104. Head Lice Which of the following is true regarding management of head lice? A. Children should be kept out of school until no visible evidence of nits is noted. B. Household members should only be treated if live lice or eggs are noted within 1 cm of the scalp. C. Head lice programs have had a significant impact on lowering the incidence of head lice. D. Cleaning of bedding has little impact on lice eradication. E. The health of those exposed is more important than the confidentiality of the child affected. The answer is B. Practice guidelines published by the American Academy of Pediatrics (AAP) state that if a case of head lice is identified, all household members should be checked, and only those with live lice or eggs within 1 cm of the scalp should be treated. It is recommended to treat family members who share a bed with the person who is infected
  • 105. Tips in answering MCQs:  In general, respect your clinical experience  If your clinical practice is different than MCQ correct answer according to the reference book . Go with the book
  • 106. Number Technique Before the diagnosis of post-traumatic stress disorder (PTSD) is made, symptoms should be present for at least A. 1 year B. 6 months C. 3 months D. 1 month E. 1 week .28
  • 107. Number Technique Before the diagnosis of post-traumatic stress disorder (PTSD) is made, symptoms should be present for at least A. 1 year B. 6 months C. 3 months D. 1 month E. 1 week
  • 108. Number /Anorexia Nervosa In young women with an eating disorder, at what point would you expect her menstrual periods to resume? A. 75% of ideal body weight B. 80% of ideal body weight C. 90% of ideal body weight D. 100% of body weight E. It is unusual for menstrual cycles to resume with any weight gain. .29
  • 109. Number /Anorexia Nervosa In young women with an eating disorder, at what point would you expect her menstrual periods to resume? A. 75% of ideal body weight B. 80% of ideal body weight C. 90% of ideal body weight D. 100% of body weight E. It is unusual for menstrual cycles to resume with any weight gain.
  • 110.
  • 111. In a healthy patient, what is the appropriate age to administer Pneumococcal vaccine? A. 50 years B. 5 years C. 75 years D. 16 years E. 65 years .30
  • 112. In a healthy patient, what is the appropriate age to administer Pneumococcal vaccine? A. 5 years B. 16 years C. 50 years D. 65 years E. 75 years
  • 113. Tips in answering MCQs: Some MCQs ask about number . Learn how to deal with numbers  Arrange the answers in order – lowest->> highest  Eliminate the lowest and highest number  Go with the general theme of family medicine practice  Chose the meaningful number
  • 114. Use of Antibiotics Which one of the following is appropriate and effective treatment for genitourinary gonorrhea in a 20-year-old male with a purulent urethral discharge? A. Amoxicillin, 3.5 g orally once. B. Ciprofloxacin, 500 mg orally once. C. Ceftriaxone, 125 mg intramuscularly once. D. Doxycycline, 100 mg 2 times daily for 3 days. E. Erythromycin, 500 mg 4 times daily for 7 days. .31
  • 115. Use of Antibiotics Which one of the following is appropriate and effective treatment for genitourinary gonorrhea in a 20-year-old male with a purulent urethral discharge? A. Amoxicillin, 3.5 g orally once. B. Ciprofloxacin, 500 mg orally once. C. Ceftriaxone, 125 mg intramuscularly once. D. Doxycycline, 100 mg 2 times daily for 3 days. E. Erythromycin, 500 mg 4 times daily for 7 days.
  • 116. Use of Antibiotics Of the following antibiotics, which one would be acceptable to use when treating penicillin-resistant S. pneumoniae otitis media? A. Azithromycin B. Clarithromycin C. Cefuroxime D. Cefaclor E. Cephalexin .32
  • 117. Use of Antibiotics Of the following antibiotics, which one would be acceptable to use when treating penicillin-resistant S. pneumoniae otitis media? A. Azithromycin B. Clarithromycin C. Cefuroxime D. Cefaclor E. Cephalexin Only five antibiotics—high-dose amoxicillin (80 mg/kg/day), amoxicillin–clavulanate (Augmentin), cefuroxime (Ceftin), cefprozil (Cefzil), and ceftriaxone (Rocephin)—have demonstrated a modest degree (60% to 80%) of clinical efficacy in the treatment of acute otitis media caused by penicillin-resistant S. pneumoniae.
  • 118. Antibiotics An 18-year-old woman presents with swelling, warmth, and spreading redness at the upper part of her ear, where she recently underwent an ear piercing. Appropriate antibiotic coverage includes A. cephalexin B. ciprofloxacin C. azithromycin D. penicillin E. tetracycline .33
  • 119. Antibiotics An 18-year-old woman presents with swelling, warmth, and spreading redness at the upper part of her ear, where she recently underwent an ear piercing. Appropriate antibiotic coverage includes A. cephalexin B. ciprofloxacin C. azithromycin D. penicillin E. tetracycline
  • 120. Depression A 14-year-old girl is diagnosed with major depressive disorder. It is her first episode and the clinical assessment is that it is mild depression. Which one of the following treatment choices is appropriate? A. Tricyclic antidepressant.. B. SSRI. C. St. John's wart. D. Cognitive-behavioral therapy. E. Lithium. .34
  • 121. Depression A 14-year-old girl is diagnosed with major depressive disorder. It is her first episode and the clinical assessment is that it is mild depression. Which one of the following treatment choices is appropriate? A. Tricyclic antidepressant.. B. SSRI. C. St. John's wart. D. Cognitive-behavioral therapy. E. Lithium.
  • 122. Alcoholism management  A 56-year-old business executive with a history of alcoholism and associated liver disease presents to your office and would like to stop drinking. In order to prevent alcohol withdrawal, you select which of the following medications? A. Lorazepam B. Clonazepam C. Diazepam D. Flurazepam E. Buspirone .35
  • 123. Alcoholism management  A 56-year-old business executive with a history of alcoholism and associated liver disease presents to your office and would like to stop drinking. In order to prevent alcohol withdrawal, you select which of the following medications? A. Lorazepam B. Clonazepam C. Diazepam D. Flurazepam E. Buspirone
  • 124. Anti depressant Medications Which of the following medications is least likely to cause sexual side effects? A. Fluoxetine (Prozac) B. Sertraline (Zoloft) C. Venlafaxine (Effexor) D. Citalopram (Celexa) E. Bupropion (Wellbutrin) .36
  • 125. Anti depressant Medications Which of the following medications is least likely to cause sexual side effects? A. Fluoxetine (Prozac) B. Sertraline (Zoloft) C.Venlafaxine (Effexor) D. Citalopram (Celexa) E. Bupropion (Wellbutrin)
  • 126. Antidepressant About how many patients in the total treated population respond only partially or don't respond at all to antidepressant monotherapy? A. 5 to 20 percent. B. 10 to 30 percent. C. 20 to 40 percent. D. 30 to 50 percent. E. 40 to 60 percent. .37
  • 127. Antidepressant About how many patients in the total treated population respond only partially or don't respond at all to antidepressant monotherapy? A. 5 to 20 percent. B. 10 to 30 percent. C. 20 to 40 percent. D. 30 to 50 percent. E. 40 to 60 percent. Placebo effect is up to 60%
  • 128. About how many patients in the total treated population respond only partially or don't respond at all to antidepressant monotherapy? A. 5 to 20 percent. B. 10 to 30 percent. C. 20 to 40 percent. D. 30 to 50 percent. E. 40 to 60 percent. Placebo effect is up to 60% Antidepressant
  • 129. Tips in studying MCQs: Concentrate on important topics in family medicine e.g.: Important Medications • Antibiotics • Antidepressant • Other psychotropic medications • Anti epilepsy medications • Anti TB • Anti ulcer medications • Antihypertensive medications • Hypoglycemic medications • Asthma medications
  • 130. Hepatitis C Infection Which one of the following is the approximate risk that a health care worker will contract hepatitis C infection following a needle stick injury contaminated by blood from a patient testing positive for hepatitis C by polymerase chain reaction? A. 0.6 percent. B. 50 percent. C. 6.1 percent. D. 16 percent. E. 26 percent. .38
  • 131. Hepatitis C Infection Which one of the following is the approximate risk that a health care worker will contract hepatitis C infection following a needle stick injury contaminated by blood from a patient testing positive for hepatitis C by polymerase chain reaction? A. 0.6 percent. B. 50 percent. C. 6.1 percent. D. 16 percent. E. 26 percent.
  • 132. Which one of the following is the approximate risk that a health care worker will contract hepatitis C infection following a needle stick injury contaminated by blood from a patient testing positive for hepatitis C by polymerase chain reaction? A. 0.6 percent. B. 50 percent. C. 6.1 percent. D. 16 percent. E. 26 percent. Hepatitis C Infection
  • 133. Urinary Tract Infections Which one of the following is the most sensitive laboratory indicator for urinary tract infections? A. Pyuria. B. Bacteriuria. C. Leukocyte esterase. D. Increased vaginal pH. E. Urine nitrite. .39
  • 134. Urinary Tract Infections Which one of the following is the most sensitive laboratory indicator for urinary tract infections? A. Pyuria. B. Bacteriuria. C. Leukocyte esterase. D. Increased vaginal pH. E. Urine nitrite. Respect your practical experience
  • 135. HIV In which one of the following situations should a health care worker be offered antiretroviral prophylaxis after an occupational exposure to a patient known to be infected with human immunodeficiency virus? A. Urine with no visible blood in it splashed into the worker's eyes. B. Needle-stick injury when recapping a clean needle. C. Intact skin contact with patient's tears. D. Accidental cut from a used sharp instrument. E. Urine with no visible blood in it splashed onto exposed skin. .40
  • 136. HIV In which one of the following situations should a health care worker be offered antiretroviral prophylaxis after an occupational exposure to a patient known to be infected with human immunodeficiency virus? A. Urine with no visible blood in it splashed into the worker's eyes. B. Needle-stick injury when recapping a clean needle. C. Intact skin contact with patient's tears. D. Accidental cut from a used sharp instrument. E. Urine with no visible blood in it splashed onto exposed skin. Common sense
  • 137. HIV Infection What is the approximate risk of (HIV) seroconversion after a needle-stick injury when the source patient has documented HIV infection? A. 0.01 to 0.05 percent. B. 0.3 to 0.5 percent. C. 1.0 to 5.0 percent. D. 25 to 30 percent. E. Nearly 100 percent. .41
  • 138. HIV Infection What is the approximate risk of (HIV) seroconversion after a needle-stick injury when the source patient has documented HIV infection? A. 0.01 to 0.05 percent. B. 0.3 to 0.5 percent. C. 1.0 to 5.0 percent. D. 25 to 30 percent. E. Nearly 100 percent.
  • 139. Needle Stick injury & Risk to the Health Care Worker  HIV  Hepatitis C  Hepatitis B  0.3 to 0.5 percent  6.1 percent  ….
  • 140. Tips in studying MCQs: Concentrate on important topics in family medicine e.g.: Important infections  TB  Hepatitis B & C  HIV  Any current epidemic disease TB
  • 141. Falls In The Elderly Which of the following is responsible for the largest proportion of falls in the elderly? A. Extrinsic (environmental) factors. B. Postural changes. C. Dizziness. D. Gait problems. E. Impaired vision. .42
  • 142. Falls In The Elderly Which of the following is responsible for the largest proportion of falls in the elderly? A. Extrinsic (environmental) factors. B. Postural changes. C. Dizziness. D. Gait problems. E. Impaired vision. Different
  • 143. Asymptomatic Bacteriuria Of the following outcomes, which one is associated with antibiotic treatment of asymptomatic bacteriuria in elderly women? A. Prevention of pyelonephritis. B. Prevention of hypertension. C. Sterilization of the urinary tract. D. Emergence of drug-resistant bacterial strains. E. Overall reduction in mortality. .43
  • 144. Asymptomatic Bacteriuria Of the following outcomes, which one is associated with antibiotic treatment of asymptomatic bacteriuria in elderly women? A. Prevention of pyelonephritis. B. Prevention of hypertension. C. Sterilization of the urinary tract. D. Emergence of drug-resistant bacterial strains. E. Overall reduction in mortality. Different
  • 145. Tips in answering MCQs: and the answer you have chosen are grammatically correct If you are not sure of the answer make intelligent guising:  If one choice is different from the rest, consider it the right answer
  • 146. Acellular Pertussis Vaccine Which one of the following statements about the acellular pertussis vaccine is true? A. It is about as effective as the whole-cell vaccine with fewer side effects. B. It is a little less effective than the whole-cell vaccine with fewer side effects. C. It is as effective as the whole-cell vaccine but does not show an appreciable difference in side effects. D. It is less effective than the whole-cell vaccine with no significant difference in side effects. E. It is less effective than the whole-cell vaccine with more side effects. .44
  • 147. Which one of the following statements about the acellular pertussis vaccine is true? A. It is about as effective as the whole-cell vaccine with fewer side effects. B. It is a little less effective than the whole-cell vaccine with fewer side effects. C. It is as effective as the whole-cell vaccine but does not show an appreciable difference in side effects. D. It is less effective than the whole-cell vaccine with no significant difference in side effects. E. It is less effective than the whole-cell vaccine with more side effects. New vaccine has same or better effects and less side effects
  • 148. Tips in answering MCQs: Examples logical thinking:  To change to any new intervention (drug, procedure, investigation or vaccine …) the effect of the new intervention must be the same or better effects and with less side effects
  • 149. Thrombocytopenic Purpura The following statement about thrombocytopenic purpura is correct? A. They present most often with nosebleeds or bleeding gums. B. They tend to be girls. C. They remit spontaneously in up to 90 percent of cases. D. They have a slightly higher risk of intracerebral hemorrhage than adults. E. They show poikilocytosis and nucleated red cells on the peripheral smear. .45
  • 150. Thrombocytopenic Purpura The following statement about thrombocytopenic purpura is correct? A. They present most often with nosebleeds or bleeding gums. B. They tend to be girls. C. They remit spontaneously in up to 90 percent of cases. D. They have a slightly higher risk of intracerebral hemorrhage than adults. E. They show poikilocytosis and nucleated red cells on the peripheral smear.
  • 151. Tips in answering MCQs: Respect your logical thinking & your clinical experience

Notas del editor

  1. Preparing for an MCQ-based exam is completely different from preparation method for all other types of exams. You need to direct your energies in the right way, otherwise you will continue to study and still get No Output!Coming straight to the point, I shall be telling you the best method (Tested and Proven by Experts) to prepare for an MCQ-based exam. So how do you prepare for an MCQ exam? I've compiled a list of tips:Revise everything, and don't try to 'question spot'. MCQ exams can cover an awful lot of material - that's the point.Make sure your knowledge is sufficiently specific. MCQs can be highly specific, so a general knowledge of an area might not be enough.However, a good broad knowledge is helpful when attempting to deduce the correct answer to a question you're unsure of.Practice, practice, practice and then do some more practice. Gather together as many questions as you can lay your hands on.Make sure your practice MCQs cover material that's relevant to your course; there's no point in learning things you won't be asked.Pile on the pressure by practicing under exam conditions. This will help you become accustomed to answering questions against the clock, and allow you to accurately judge how much time you can allow for each question.Get hold of the Exam Answering Sheets or similar sheets and practice in real time.If you try to recreate exam conditions when you practice, you'll find your memory will be triggered under similar conditions in the exam hall.Revise the things you don't know. Sounds obvious, but you're actually more likely to spend time on topics you already know - it's more fun and can be better for morale. Be brave, and tackle your weakest areas (neurology?!) first.Revise with friends; it's easier to stay motivated - and much more fun! You can share knowledge this way too. Make sure you revise with friends of a similar ability - there's nothing more depressing than finding everyone else knows more than you do. Alternatively, if you relish a challenge, try revising with the best and brightest - they may inspire you to raise your game.On the day of the exam, make sure you're clear about the exam format: how many questions are there? How long have you got? Will there be negative marking?A common mistake made by the MCQ novice is believing that MCQs are only a test of memory, and if you have perfect recall of everything you've been taught, you can't fail. However, although they do test your ability to recall facts, MCQs can also test your ability to interpret information and apply knowledge to both familiar and unfamiliar scenarios. Success in MCQ tests is not simply a case of memorising everything you've been taught.Some of the most common mistakes committed by students while preparing for MCQ-based exams are as follows:-Do not try to memorize everything. Learn with logic and relate information between different books on the same topic.The most difficult way to prepare for MCQ based exam is to study alone. Studying in a group of like-minded students can be very helpful.Go through the entire course of study. Do Not leave anything on choice.Even if you can not study everything multiple times, you must go through it at least once and discuss with friends. You will surely recall it, if you see it in exams.Instead of studying hard, you must Study Smart for an MCQ based exam.
  2. Infectious mononucleosis: A specific viral infection (with the Epstein-Barr virus) in which there is an increase of white blood cells that are mononuclear (with a single nucleus) "Mono" and "kissing disease" are popular terms for this very common illness caused by the Epstein-Barr virus (EBV). By the time most people reach adulthood, an antibody against EBV can be detected in their blood meaning they have been infected with EBV. The illness is less severe in young children. The infection can be spread by saliva. The incubation period is 4 to 8 weeks.
  3. The answer is A. Breast-feeding is encouraged for all mothers.Currently, as many as 50% of mothers (especially those in higher socioeconomic groups) are breast-feeding. In most cases, theinfant should feed at each breast for 8 to 15 minutes every 2 to 3hours after birth and can be started immediately after delivery.Colostrum, a yellowish fluid excreted from the breast immediately after delivery, contains important antibodies, high calories, andhigh proteins, as well as other nutrients and helps stimulate the passage of meconium. Some studies have shown that delayingbreast-feeding, trying to quantify amounts of feeding with prefeed and postfeed weights, and providing infant formula decrease thepercentage of women who breast-feed by discouraging the practice. Breast-feeding is usually adequate nutrition for 6 to 9months. If mothers develop sore nipples, they should be counseled with regard to proper positioning of the baby's mouth on the breast. The production of a lubricant from Montgomery's glands occurs and helps protect the breast from excessive drying. Typically, breast-fed infants require more frequent feedings than bottle-fed infants. Breast-feeding should occur based on demand rather than by the clock. Breast engorgement can be avoided with more frequent feedings or manual expression of excessive milk production with breast pumps. New mothers should initiate breastfeeding as soon as possible after giving birth. When mothers initiate breast-feeding within one-half hour of birth, the baby's suckling reflex is strongest, and the baby is more alert. Early breast-feeding is associated with fewer nighttime feeding problems and better mother-infant communication. Babies who are put to breast earlier have been shown to have higher core temperatures and less temperature instability
  4. Answer is A. . FTT may be a result of organic causes, nonorganic causes, or both. Nonorganic causes predominate. Nonorganic FTT includes psychologic FTT (maternal deprivation), child neglect, lack of education regarding feeding, and errors in feeding. Nonorganic FTT is most often attributable to maternal deprivation (as in this case) or lack of a nurturing environment at home. Organic FTT is caused most commonly by a medical condition impairing the child’s ability to take in, absorb, or metabolize adequate calories.
  5. Answer is A. . FTT may be a result of organic causes, nonorganic causes, or both. Nonorganic causes predominate. Nonorganic FTT includes psychologic FTT (maternal deprivation), child neglect, lack of education regarding feeding, and errors in feeding. Nonorganic FTT is most often attributable to maternal deprivation (as in this case) or lack of a nurturing environment at home. Organic FTT is caused most commonly by a medical condition impairing the child’s ability to take in, absorb, or metabolize adequate calories.
  6. The Answerv is B. Bone age determination can distinguish between the two most common causes of short stature: familial short stature and constitutional delay of growth. Children with familial short stature have normal bone ages. Constitutional delay of growth, which is really a delay in reaching ultimate height and sexual maturation, presents with delayed bone age and delayed sexualmaturation. Hypothyroidism and growth hormone deficiency usually present with a delayed bone age
  7. The Answerv is B. Bone age determination can distinguish between the two most common causes of short stature: familial short stature and constitutional delay of growth. Children with familial short stature have normal bone ages. Constitutional delay of growth, which is really a delay in reaching ultimate height and sexual maturation, presents with delayed bone age and delayed sexualmaturation. Hypothyroidism and growth hormone deficiency usually present with a delayed bone age
  8. The answer is A. The U.S. Preventive Services Task Force (USPSTF) recommends screening to detect amblyopia, strabismus, and defects in visual acuity in children younger than age 5 years. The USPSTF found no evidence of harm associated with screening, and concluded that the benefits of screening are likely to outweigh any potential harms. The most common causes of visual impairment in children are (1) amblyopia and its risk factors and (2) refractive error not associated withamblyopia.
  9. The answer is A. The U.S. Preventive Services Task Force (USPSTF) recommends screening to detect amblyopia, strabismus, and defects in visual acuity in children younger than age 5 years. The USPSTF found no evidence of harm associated with screening, and concluded that the benefits of screening are likely to outweigh any potential harms. The most common causes of visual impairment in children are (1) amblyopia and its risk factors and (2) refractive error not associated withamblyopia.
  10. The answer is C. ELISA serology testing, although convenient and commonly used, is not reliable to determine successful eradication of H. pylori.Antibodies titers are slow to decline and can therefore lead to many false-positive results even after successful treatment. Serology testing is appropriate for patients never treated for the organism in the past. Steiner stain of gastric biopsy specimen and culture require invasive endoscopy but are sensitive and specific for detection of persistence of H. pyloriafter treatment. Stool antigen and urea breath tests are also accurate tests to check for persistence of infection. Stool tests are more convenient because they are office based compared with the urea breath test, which requires special equipment not usually available in the office
  11. The vast majority of aneurysms are asymptomatic. However, as abdominal aortic aneurysms expand, they may become painful and lead to pulsating sensations in the abdomen or pain in the chest, lower back, or scrotum.[5] The risk of rupture is high in a symptomatic aneurysm, which is therefore considered an indication for surgery. The complications include rupture, peripheral embolization, acute aortic occlusion, and aortocaval (between the aorta and inferior vena cava) or aortoduodenal (between the aorta and the duodenum)fistulae. On physical examination, a palpable abdominal mass can be noted. Bruits can be present in case of renal or visceral arterialstenosis.[6]The clinical manifestation of ruptured AAA usually includes excruciating pain of the lower back, flank, abdomen and groin. The bleeding usually leads to a hypovolemic shock with hypotension, tachycardia, cyanosis, and altered mental status. The mortality of AAA rupture is up to 90%. 65–75% of patients die before they arrive at hospital and up to 90% die before they reach the operating room.[7] The bleeding can be retroperitoneal or intraperitoneal, or the rupture can create an aortocaval or aortointestinal (between the aorta and intestine) fistula.[8] Flank ecchymosis (appearance of a bruise) is a sign of retroperitoneal hemorrhage, and is also called Grey Turner's sign
  12. The vast majority of aneurysms are asymptomatic. However, as abdominal aortic aneurysms expand, they may become painful and lead to pulsating sensations in the abdomen or pain in the chest, lower back, or scrotum.[5] The risk of rupture is high in a symptomatic aneurysm, which is therefore considered an indication for surgery. The complications include rupture, peripheral embolization, acute aortic occlusion, and aortocaval (between the aorta and inferior vena cava) or aortoduodenal (between the aorta and the duodenum)fistulae. On physical examination, a palpable abdominal mass can be noted. Bruits can be present in case of renal or visceral arterialstenosis.[6]The clinical manifestation of ruptured AAA usually includes excruciating pain of the lower back, flank, abdomen and groin. The bleeding usually leads to a hypovolemic shock with hypotension, tachycardia, cyanosis, and altered mental status. The mortality of AAA rupture is up to 90%. 65–75% of patients die before they arrive at hospital and up to 90% die before they reach the operating room.[7] The bleeding can be retroperitoneal or intraperitoneal, or the rupture can create an aortocaval or aortointestinal (between the aorta and intestine) fistula.[8] Flank ecchymosis (appearance of a bruise) is a sign of retroperitoneal hemorrhage, and is also called Grey Turner's sign
  13. The vast majority of aneurysms are asymptomatic. However, as abdominal aortic aneurysms expand, they may become painful and lead to pulsating sensations in the abdomen or pain in the chest, lower back, or scrotum.[5] The risk of rupture is high in a symptomatic aneurysm, which is therefore considered an indication for surgery. The complications include rupture, peripheral embolization, acute aortic occlusion, and aortocaval (between the aorta and inferior vena cava) or aortoduodenal (between the aorta and the duodenum)fistulae. On physical examination, a palpable abdominal mass can be noted. Bruits can be present in case of renal or visceral arterialstenosis.[6]The clinical manifestation of ruptured AAA usually includes excruciating pain of the lower back, flank, abdomen and groin. The bleeding usually leads to a hypovolemic shock with hypotension, tachycardia, cyanosis, and altered mental status. The mortality of AAA rupture is up to 90%. 65–75% of patients die before they arrive at hospital and up to 90% die before they reach the operating room.[7] The bleeding can be retroperitoneal or intraperitoneal, or the rupture can create an aortocaval or aortointestinal (between the aorta and intestine) fistula.[8] Flank ecchymosis (appearance of a bruise) is a sign of retroperitoneal hemorrhage, and is also called Grey Turner's sign
  14. The answer is D. Vomiting with nausea is not a solid indication for neuroimaging during management of a migraine headache. Although some have said that vomiting with or without nausea is an indication for neuroimaging, actually nausea and vomiting associated with migraine is relatively common and not of great concern except when there is no history of these symptoms being associated with headaches or if there be anything “different” about the headache that presents with nausea or vomiting. Certain indications for neuroimaging with migraine include seizures, prolonged aura, onset of headaches over the age of 50 years, headache worsening with movement, symptoms of systemic illness (e.g., fever), and any neurological signs that were not present at baseline
  15. Tips: Number tips The answer is C. Menstruation usually resumes in women affected with anorexia when the patient approaches 90% of ideal body weight. Mehler PS. Diagnosis and care of patients with anorexia nervosa in the primary care setting 
  16. Tips: Number tips The answer is C. Menstruation usually resumes in women affected with anorexia when the patient approaches 90% of ideal body weight. Mehler PS. Diagnosis and care of patients with anorexia nervosa in the primary care setting 
  17. The answer is A. An evidence-based guideline from the American Society of Addiction Medicine recommends benzodiazepines as a first-line agent for the treatment of alcohol withdrawal. The guideline notes that although agents with a longer duration of action may provide fewer breakthrough symptoms, those with a shorter duration of action, uch as lorazepam (Ativan), may be preferred when there is concern about prolonged sedation (e.g., in patients with significant comorbidities or liver disease).
  18. The answer is E. Sexual side effects, usually delayed ejaculation or anorgasmia, may occur in both men and women who are taking SSRIs and venlafaxine (Effexor). Treatment consists of several options: reducing the dosage, switching to another agent, or adding another agent to overcome the sexual side effects. Sexual dysfunction typically resolves within 1 to 3 days after discontinuation of the antidepressant and returns on reintroduction. Recovery after withdrawal from fluoxetine (Prozac) may occur within 1 to 3 weeks. Studies suggest that the addition of bupropion (Wellbutrin) may decrease sexual side effects. Ables AZ, Baughman AO III. Antidepressants: update on new agents and indications